Facultair Overlegorgaan Sociale Wetenschappen … · Web viewSolidariteit op de werkvloer maakt...

57
DAAN DELESPAUL 2017-2018 SAMENVATTING READER POLITIEKE SOCIOLOGIE 1. A. De Tocqueville (1835): De la Démocratie en Amérique (fragment) Verenigingen zijn een middel voor de minderheid om de meerderheid te controleren. Ze leren hun leden ook civic skills aan om mee te doen aan het democratisch proces en vormen zo een link tussen burger en politiek. Samenvatting De VSA kennen veel verenigingen. Deze vormen een school voor waarden en normen die vereist zijn voor het dagelijks samenleven. Daarom is een recht van vereniging, geholpen door vrijheid van meningsuiting en de mogelijkheid overal fysiek samen te komen ook essentieel voor de politiek. Verenigingen ontstaan door mensen met eenzelfde mening die samen een fractie vormen. Binnen deze overheden ontstaan dan regeringen (=partijen) die geen formele macht hebben, maar haar met kritiek wel langs de zijlijn bestrijden. Het is immers belangrijk dat een minderheid het de tirannie van de meerderheid moeilijk maakt door middel van kritiek en de vrijheid van meningsuiting: een meerderheid kan de staat immers totaal overnemen. Nergens zijn verenigingen dan ook harder nodig om het despotisme te doorbreken dan in een democratie. Naast de natuurlijke vrijheid is het recht op zelfbeschikking en dat om samen te werken van belang. Vrijheid zit reeds in de Amerikaanse samenleving en overheid ingebakken: iedereen heeft eenzelfde doel maar met andere middelen. Men verliest zo zijn ratio en moet zijn eigen wil niet opgeven voor de staat, maar gebruikt hem juist in het voordeel van de staat. Amerikanen hebben het gebruik zich in het dagelijks leven te verenigen, niet enkel voor sociale, maar ook politieke doeleinden. Daarom is het ook belangrijk social skills aan te leren om macht te verwerven binnen de staat. De samenleving neemt immers een steeds belangrijkere rol op in het besluitvormingsproces. Maar samenwerken heeft nog een voordeel het zorgt voor individuele en maatschappelijke vooruitgang. Het probleem is dat het aantal leden binnen (politieke) verenigingen steeds afneemt, wat zorgt dan nog voor linkage tussen politiek en burger? De eerste verklaring voor het afnemen van vertrouwen in het politiek stelsel en de ledenaantallen bij verenigingen is dat burgers steeds individualistischer worden en daarom niet langer participeren. Deze visie van Putnam wordt ook wel ‘civic decline’ genoemd. Een andere visie die door Norris en Inglehart wordt aangehangen, denk dan weer vooral dat de waarden en de participatie eerder op een andere manier geuit worden. Burgers zijn immers steeds kritischer en mondiger en anderzijds liggen de verwachtingen ten aanzien van de politiek ook helemaal elders. 1

Transcript of Facultair Overlegorgaan Sociale Wetenschappen … · Web viewSolidariteit op de werkvloer maakt...

Page 1: Facultair Overlegorgaan Sociale Wetenschappen … · Web viewSolidariteit op de werkvloer maakt deel uit van een ruimere sociale solidariteit 2) Kerk en religiositeit: Georganiseerde

DAAN DELESPAUL 2017-2018

SAMENVATTING READER POLITIEKE SOCIOLOGIE

1. A. De Tocqueville (1835): De la Démocratie en Amérique (fragment)

Verenigingen zijn een middel voor de minderheid om de meerderheid te controleren. Ze leren hun leden ook civic skills aan om mee te doen aan het democratisch proces en

vormen zo een link tussen burger en politiek.

Samenvatting

De VSA kennen veel verenigingen. Deze vormen een school voor waarden en normen die vereist zijn voor het dagelijks samenleven. Daarom is een recht van vereniging, geholpen door vrijheid van meningsuiting en de mogelijkheid overal fysiek samen te komen ook essentieel voor de politiek. Verenigingen ontstaan door mensen met eenzelfde mening die samen een fractie vormen. Binnen deze overheden ontstaan dan regeringen (=partijen) die geen formele macht hebben, maar haar met kritiek wel langs de zijlijn bestrijden. Het is immers belangrijk dat een minderheid het de tirannie van de meerderheid moeilijk maakt door middel van kritiek en de vrijheid van meningsuiting: een meerderheid kan de staat immers totaal overnemen. Nergens zijn verenigingen dan ook harder nodig om het despotisme te doorbreken dan in een democratie.

Naast de natuurlijke vrijheid is het recht op zelfbeschikking en dat om samen te werken van belang. Vrijheid zit reeds in de Amerikaanse samenleving en overheid ingebakken: iedereen heeft eenzelfde doel maar met andere middelen. Men verliest zo zijn ratio en moet zijn eigen wil niet opgeven voor de staat, maar gebruikt hem juist in het voordeel van de staat. Amerikanen hebben het gebruik zich in het dagelijks leven te verenigen, niet enkel voor sociale, maar ook politieke doeleinden. Daarom is het ook belangrijk social skills aan te leren om macht te verwerven binnen de staat. De samenleving neemt immers een steeds belangrijkere rol op in het besluitvormingsproces. Maar samenwerken heeft nog een voordeel het zorgt voor individuele en maatschappelijke vooruitgang.

Het probleem is dat het aantal leden binnen (politieke) verenigingen steeds afneemt, wat zorgt dan nog voor linkage tussen politiek en burger? De eerste verklaring voor het afnemen van vertrouwen in het politiek stelsel en de ledenaantallen bij verenigingen is dat burgers steeds individualistischer worden en daarom niet langer participeren. Deze visie van Putnam wordt ook wel ‘civic decline’ genoemd. Een andere visie die door Norris en Inglehart wordt aangehangen, denk dan weer vooral dat de waarden en de participatie eerder op een andere manier geuit worden. Burgers zijn immers steeds kritischer en mondiger en anderzijds liggen de verwachtingen ten aanzien van de politiek ook helemaal elders.

Verdere samenvatting:

Tocqueville turns to what would be called today special interest groups and how they try to influence politics. These are private groups, more numerous in America than in other places. This is because Americans are taught from infancy to be self-reliant and not to depend upon social authority. Private societies are formed against alcohol consumption or to promote public safety, morality, commerce, and religion.  The right to associate in groups like this goes along with freedom of the press. These associations are granted the right to meet and to have headquarters. They may also combine to choose political delegates to represent them. The power of association is unlimited and provides a minority check to the majority. This right was imported

1

Page 2: Facultair Overlegorgaan Sociale Wetenschappen … · Web viewSolidariteit op de werkvloer maakt deel uit van een ruimere sociale solidariteit 2) Kerk en religiositeit: Georganiseerde

DAAN DELESPAUL 2017-2018

from England, and Tocqueville thinks it is a right sorely needed in a democracy, while in an aristocracy, the nobles and wealthy are natural associations to check the abuses of power. 

Tocqueville gives examples of special interest groups, such as temperance (anti-alcohol) leagues or groups for and against the tariff. The tariff was an import tax on goods from Europe to encourage local production and consumption, but many argued that free trade was better for the economy. These “associations” or special interest groups are almost a nation within a nation, he notes, minorities who propose laws or elect delegates for their own causes. This open lobbying for minority causes provides a check to the tyranny of a majority. In Europe, however, the association is seen as a weapon, not for discussion, but to produce action. European associations usually say they represent the majority that is not being heard, while in America, associations are largely minority positions.  American special interest groups may or may not be trying to take action. They generally have a more peaceful face in that they are hoping to convince others of their position. In America, minority associations and opinions are in the open, he says, and therefore, do not turn into conspiracies against the government. They represent a safety valve.

2. M. Hooghe (2003): Sociaal Kapitaal in Vlaanderen (p. 21-50)

Definitie politieke cultuur:

Houdingen van de burgers ten opzichte van het politiek systeem (principes, instellingen, politiek personeel, beleid) en hun visie op hun eigen rol daarin: “political efficacy” (onderscheid tussen (1)

internal political efficacy (2) external political efficacy)

(De tekst bouwt verder op de notie ‘politieke cultuur’. Dit heeft culturele infrastructuur nodig in de vorm van ‘sociaal kapitaal’)

Inleiding: tussen participatie en cohesie De afgelopen jaren is er een grotere aandacht voor de dalende politieke participatie gekomen. Volgens Amerikaanse socioloog Robert Putnam zorgt de afwezigheid van sociaal kapitaal immer voor een disfunctionele maatschappij. Sociaal kapitaal bestaat uit aanwezigheid van netwerken tussen burgers, verspreiden van wederkerigheid en de aanwezigheid van vertrouwen. Coleman meent dat het de gesloten (dus met sancties) en homogene netwerken zijn die een cultuur helpen. Open netwerken dan weer voor de verdeling van hulpbronnen. Colemans visie baseert zich dan ook op een individueel niveau, terwijl dat van Putnam zich eerder op het collectieve richt.

De centrale vraag bij Putnam is hoe een democratische samenleving functioneert. Het punt bij een democratie is dat ze maar een meerwaarde creëert bij het actief participeren van alle burgers in de besluitvorming. Hier sluit Putnam aan bij het begrip ‘politieke cultuur’ van Almond en Verba en ‘sociaal kapitaal’ van Bourdieu. Hoe kunnen nu verbondenheid en sociale orde blijvend worden gehandhaafd in een geïndividualiseerde en geseculeerde maatschappij?

Actieve burgersSociaal kapitaal wordt gecreëerd door de actieve betrokkenheid van de burger in de samenleving. Vroeg was dit nochtans een huiveringwekkende gedachte: het zou immers voor instabiliteit zorgen. Het keerpunt kwam echter na ‘The Civic Culture’ van Almond en Verba uit 1963. In dat boek verzet men zich tegen de klassieke visie dat de sterkte van de instellingen en de juridische structuur de orde in een samenleving bepaalt, maar poneert men daarentegen dat het de houding van de burgers ten aanzien van die politiek is. Dat uit zich niet enkel actief, maar ook passief, bijvoorbeeld in de houding ten aanzien van wetten en vertrouwen. Dit leidde men af uit de dekolonisering van die tijden. Aan de andere kant wensen burgers ook inspraak en is een participatief luik ook essentieel. Een goede politieke cultuur balanceert dus tussen inspraak, voice en rationaliteit, traditionaliteit, vertrouwen en loyaliteit.

2

Page 3: Facultair Overlegorgaan Sociale Wetenschappen … · Web viewSolidariteit op de werkvloer maakt deel uit van een ruimere sociale solidariteit 2) Kerk en religiositeit: Georganiseerde

DAAN DELESPAUL 2017-2018

In zulke politieke cultuur spelen verenigingen een belangrijke rol. Niet alleen zorgen ze met socialisatie van de democratische waarden ten aanzien van hun leden een trouw publiek, ze geven hen ook empowerment door individuen te verenigen en hen macht te geven.

Sociaal kapitaal: een functionalistische benaderingDe centrale vraag bij Putnams studie over sociaal kapitaal is hoe individuele actoren in staat worden gesteld collectieve actie met anderen te ontwikkelen. Een samenleving functioneert beter als het makkelijker is samenwerkingsverbanden met elkaar aan te gaan. Hij let daarbij echter ook op ‘the dark side of social capital’: een te sterke verwevenheid met de eigen organisatie maakt een individu los van de maatschappij. Een verschil met Bourdieu is dat Putnam het kapitaal op een macro-niveau ziet: door samenwerking ontstaat iets nieuws. Bourdieu ziet de vereniging eerder als een micro-eigenschap: het individu verwerft goederen door in een vereniging aanwezig te zijn, zo zorgt sociaal kapitaal voor een herverdeling. (?)

Bij Coleman verwijst sociaal kapitaal naar de hulpbronnen die een individu heeft om binnen een sociaal systeem te handelen. Hij wijst hierbij op het belang van embeddedness: de context bepaalt ten dele de beslissingen van het individu. Om voordelen te halen gebruikt de mens dit sociaal kapitaal als een homo economicus om zijn eigen voordeel te halen. Dit doet hij in netwerken (zoals de wet). Daarom is dit ook een functionalistische visie op sociaal kapitaal.

De micro- en macrovisie op sociaal kapitaal sluiten elkaar echter niet uit. Ook Putnam sluit niet uit dat individuen hun voordeel halen uit sociale netwerken, maar daar ligt zijn interesse niet; hij wil weten hoe deze collectiviteiten hun gemeenschappelijk doel bereiken. Bovendien komen deze sociale netwerken vaak ook meerdere actoren ten goede. Als je buurvrouw oplet of er geen dieven binnenkomen heeft waarschijnlijk de hele buurt daar baat bij. Sociaal kapitaal is dan ‘alles wat het samenwerken makkelijker maakt’, namelijk om de doelen van de collectiviteiten te bereiken. Dit valt uiteen in vertrouwen, netwerken en wederkerigheid.

Structurele en culturele componentenColeman deelt sociaal kapitaal op in twee componenten: het structurele, dat de integratie in netwerken betreft, en het culturele dat de heersende normen en waarden binnen een netwerk betreft. Wat dit structurele niveau betreft zijn twee zaken van belang: vertrouwen en wederkerigheid. De positie van vertrouwen is eerder twijfelachtig: is het wel vertrouwen dat voor vereniging zorgt, of is dit andersom? Wederkerigheid gaat ervan uit dat men iets terugkrijgt en als een gelijke wordt behandeld.

Vertrouwen vereist een cultureel component: met individuen die men kent of voorspelbaar acht zal men eerder samenwerken. Gemarginaliseerde groepen komen zo minder in het participatieproces terecht en zullen anderzijds zelf eveneens geen vertrouwen ontwikkelen door verenigingen. Op deze wijze verklaart vertrouwen een machtsongelijkheid in het systeem van sociaal kapitaal. Ook netwerking is daarom belangrijk voor het culturele component.

Kritiek PutnamVolgens de critici is er niet zoiets als een homogene samenleving met een algemeen vertrouwen. Bovendien is een democratie juist uitgebouwd op basis van wantrouwen in de machthebbers. Het opnemen van een variabele vertrouwen is dan ook moeilijk te rechtvaardigen. Hiertegen brengt Putnam op dat het niet de ‘bonding’, maar de ‘bridging’ netwerken zijn die voor sociaal kapitaal zorgen. Dat neemt niet weg dat netwerken de handelingsvrijheid nog steeds inperken en culturele hegemonie versterken.

Causale verbandenAls verklaring voor de verschillende vertrouwenswaarden wijst Putnam op de ‘path dependency’: zo hebben noordelijke en protestantse landen meer vertrouwen in hun cultuur door hun verleden. Ook op microniveau zorgen o.m. tijdsdruk, de introductie van de tv en een ander waardenpatroon voor een minder vertrouwen. Hierbij negeert hij de belangrijke rol van de overheid bij het vestigen van vertrouwen. De vraag blijft bovendien nog steeds of verenigingen het vertrouwen wel helpen (de Tocqueville) en zo ja welke verenigingen dan?

3

Page 4: Facultair Overlegorgaan Sociale Wetenschappen … · Web viewSolidariteit op de werkvloer maakt deel uit van een ruimere sociale solidariteit 2) Kerk en religiositeit: Georganiseerde

DAAN DELESPAUL 2017-2018

3. G. Almond & S. Verba (1963): The Civic Culture. Political Attitudes and Democracy in Five

Nations (p. 1-35)Na Wereldoorlog II rijst de vraag hoe men nieuwe stabiele democratieën kan creëren. In hun basiswerk ‘The Civic Culture’ plaatsen Gabriel Almond en Sidney Verba het begrip politieke cultuur naar voren als basis voor een stabiele democratie. Het is volgens hun dan ook niet van belang hoe krachtig de instellingen functioneren.

Een benadering tot Politieke cultuurHet geloof van de Verlichting in de onvermijdelijke triomf van de menselijke rede en vrijheid is tijdens de voorbije eeuw tweemaal geschokt. De ontwikkeling van Fascisme en Communisme in Europa na WOI deed serieuze vragen rijzen over de onvermijdelijkheid van de democratie in het Westen. Na WOII resen er vragen over de toekomst van de democratie op wereldschaal. Culturele veranderingen hebben een nieuwe betekenis gekregen in de wereldgeschiedenis. Ze gebeuren nu op wereldschaal en aan een steeds sneller tempo.

De centrale vraag voor het publiek beleid in de volgende decennia zal de vraag naar de inhoud van deze opkomende wereldcultuur zijn. We hebben al een gedeeltelijk antwoord op deze vraag en konden dit voorspellen vanuit onze kennis van de processen van culturele verspreiding. De fysieke goederen en de wijze van hun productie (en de technologie die aan de basis ervan ligt) blijken de minste moeilijkheden bij hun verspreiding te ondervinden. Ook het model van rationele bureaucratie (die zorgt voor transport, communicatie en onderwijs in ruil voor taxatie, regelgeving en administratie) verspreide relatief gemakkelijk. Hoewel de niet-Westerse wereld nog ver staat van het succesvol ontwikkelen van een industriële technologie en een efficiënte bureaucratie, bestaat er weinig twijfel over dat ze deze instellingen wilt en dat ze er een notie van heeft.

Het probleem van de inhoud van de opkomende wereldcultuur is zijn politiek karakter. Eén aspect van deze nieuwe politieke wereldcultuur is zichtbaar: het zal gaan om een politieke cultuur van participatie. Wat de wijze van participatie zal zijn echter is onduidelijk. Er zijn twee verschillende modellen van de moderne participatorische staat, de democratische en de totalitaire. De democratische staat biedt aan de gewone man de mogelijkheid om deel te nemen aan het politieke beslissingsproces; de totalitaire staat biedt hem de rol van deelnemend onderwerp (participant subject) aan.

Wanneer er voor het democratisch model gekozen wordt is er meer nodig dan de formele instituties van democratie (algemeen stemrecht, de politieke partij...). Deze komen immers net zo goed voor in het totalitaire model zei het dan in een formele niet functionele zin. De democratische vorm heeft ook een politieke cultuur nodig die er consistent mee is. Het overbrengen van de politieke cultuur in nieuwe landen stuit op behoorlijke problemen. Hiervoor zijn twee principale redenen.

De eerste reden heeft te maken met de aard van de democratische cultuur zelf. De idee van democratie werkt inspirerend, maar de implementatie ervan is geen vanzelfsprekendheid. In het Westen staat de democratie eigenlijk nog maar pas in zijn steigers en begint de sociale wetenschap nog maar pas aan de ontcijfering van de opererende karakteristieken van het democratisch beleid zelf.

De tweede reden gaat over de objectieve problemen die deze naties confronteren. Met verouderde technologieën en sociale systemen worden ze aangetrokken door de macht van de technologische en wetenschappelijke revoluties. Dat dit voor enorme cultuurschokken zorgt spreekt voor zich.

The civic cultureDe civic culture is geen moderne cultuur, maar een cultuur die moderniteit met traditie combineert. Groot-Brittannië is een voorbeeld van hoe een dergelijke cultuur kan ontwikkelen. Die kwam tot stand door het mercantiele karakter van de samenleving.

Types van politieke cultuurDe term “politieke cultuur” wordt in de tekst gebruikt omwille van twee redenen. Ten eerste verwijst de term “politieke cultuur” naar de specifieke politieke oriëntaties – houdingen tegenover het politieke systeem en zijn verschillende delen - en houdingen ten opzichte van het zelf in het systeem. Ten tweede zorgt het concept van politieke cultuur ervoor dat we de conceptuele kaders en benaderingen van de antropologie, sociologie en psychologie kunnen gebruiken.

4

Page 5: Facultair Overlegorgaan Sociale Wetenschappen … · Web viewSolidariteit op de werkvloer maakt deel uit van een ruimere sociale solidariteit 2) Kerk en religiositeit: Georganiseerde

DAAN DELESPAUL 2017-2018

Het concept cultuur wordt hier in één specifieke betekenis gebruikt namelijk die van de psychologische oriëntatie ten opzichte van sociale objecten. Wanneer er gesproken wordt over de politieke cultuur van een samenleving, dan wordt er verwezen naar het politiek systeem geïnterneerd in de cognities, gevoelens en evaluaties van zijn populatie.

De politieke cultuur van een natie is de specifieke verspreiding van oriëntatieschema’s betreffende de politieke objecten onder de leden van de natie. Voor we die verspreiding kunnen onderscheiden moeten we eerst de verschillende wijzen van de politieke oriëntaties en de politieke objecten definiëren en specifiëren.

We onderscheiden 3 types van politieke oriëntatie. Ten eerste “cognitieve oriëntatie” dit is de wetenschap en het geloof over het politieke systeem, zijn rollen en de verplichtingen van deze rollen, zijn inputs en outputs; ten tweede “affectieve oriëntatie” of gevoelens over het politieke systeem, zijn rollen, personeel en prestaties; en ten derde “evaluatieve oriëntatie”, de beoordelingen en de opinies over politieke objecten.

We onderscheiden 4 klassen van objecten van politieke oriëntatie. Ten eerste de oriëntatie tegenover het “algemeen” politieke systeem. Ten tweede onderscheiden we oriëntaties tegenover het “zelf” als politieke actor. Ten derde onderscheiden we de oriëntaties tegenover input objecten in het politieke proces (vb. politieke partijen, belangengroepen) en ten slotte de oriëntaties ten opzichte van output objecten: het administratieve proces). Het onderscheid tussen deze laatste twee is niet altijd even duidelijk te maken, toch kiezen de auteurs voor deze indeling. De output en de input objecten kunnen elk onderverdeeld worden in drie klassen van objecten: (1) specifieke rollen of structuren, (2) verantwoordelijke rollen, (3) bepaalde openbaar beleid, beslissingen of het afdwingen van beslissingen.

We kunnen wat we tot nu toe gezegd hebben over individuele oriëntaties tegenover beleid consolideren in een simpele 3 × 4 matrix:

Algemeen politiek systeem (1)

Input objecten (3)

Output objecten (4)

Zelf als politieke actor (2)

Cognities

Gevoel

evaluatie

Het karakteriseren van de politieke cultuur van een natie betekent het invullen van zulk een matrix voor een representatieve staal van zijn populatie. De politieke cultuur wordt de frequentie van verschillende cognitieve, affectieve en evaluatieve oriëntaties tegenover het politieke systeem in het algemeen, zijn input en output zijden, en het zelf als politieke actor.

Algemeen politiek systeem

Input objecten Output objecten Zelf als politieke actor

Parochiaal 0 0 0 0

Subject 1 0 1 0

Deelnemer 1 1 1 1

Parchial Political Culture. Dit zijn samenlevingen zonder gespecialiseerde politieke rollen: “hoofdman”-schap, “leider”-schap, “sjamaan”-schap zijn diffuse politieke-economische-religieuse rollen en voor de leden van deze gemeenschappen is de politieke oriëntatie tegenover deze rollen niet gescheiden van hun religieuze en sociale oriëntaties. De burger verwacht niets van het politieke systeem in termen van verandering. Het zuivere “parochialism” komt meer waarschijnlijk voor in simpeler traditionele systemen waar politieke specialisatie minimaal is, maar bv. ook het Ottomaanse Rijk valt er onder.

The Subject Political Culture. Het “subject” is zich bewust van een gespecialiseerde overheids-autoriteit; hij heeft er een affectieve relatie mee, en evalueert het als legitiem of niet. Maar de relatie ten opzichte van het systeem is enkel op een algemeen niveau (niet tgo. het “zelf”), en tegenover de output, administratieve zijde van het politieke systeem (niet tgo. de input zijde). Het gaat in essentie om een passieve relatie. Monarchieën (let wel geen parlementaire monarchieën) zijn een voorbeeld van dit soort van samenlevingen.

5

Page 6: Facultair Overlegorgaan Sociale Wetenschappen … · Web viewSolidariteit op de werkvloer maakt deel uit van een ruimere sociale solidariteit 2) Kerk en religiositeit: Georganiseerde

DAAN DELESPAUL 2017-2018

The Participant Political Culture. De leden van de gemeenschap hebben de neiging zichzelf expliciet te oriënteren naar het systeem als een geheel en naar zowel de politieke (input) als de administratieve (output) structuren en processen toe. Individuele leden van het participatie beleid kunnen voor of tegen de vele klassen van politieke objecten georiënteerd zijn. Ze hebben de neiging om zich te oriënteren naar een “activist” rol van het zelf in het beleid, hoewel hun gevoelens en evaluatie van zo een rol kan variëren van aanvaarding tot verwerping.

Er zijn twee opmerkingen die we hierbij moeten maken:

Ten eerste gaat deze drievoudige classificatie van politieke culturen er niet vanuit dat de ene oriëntatie de anderen vervangt. De verschillende politieke culturen bestaan tegelijkertijd en passen zich ook aan elkaar aan. De leden van een gemeenschap kunnen lid zijn van meerdere politieke culturen tezelfdertijd.

Ten tweede impliceert onze classificatie geen homogeniteit of uniformiteit van politieke cultuur. In participerende culturen kunnen bijvoorbeeld subjecten en parochianen voorkomen.

Er zijn dus twee aspecten van culturele heterogeniteit of culturele “mix”. De “burger” is een bepaalde mix van participant, subject en parochiale oriëntaties, en de “civic culture” is een bepaalde mix van burgers, subjecten en parochianen.

De politieke cultuur kan al dan niet congruent zijn met de structuren van het politieke systeem. In het algemeen zou een parochiaal, subject, of participant cultuur het meest congruent zijn met, respectievelijk, een traditionele politieke structuur, een gecentraliseerde autoritaire structuur, en een democratische politieke structuur. Politieke systemen veranderen en we kunnen rechtsmatig aannemen dat cultuur en structuur vaak incongruent zijn. Meer nog in deze decennia (het artikel dateert van 1963!) van snelle culturele veranderingen, zijn de meest voorkomende politieke systemen diegene die falen in het behouden van congruentie, of die van het ene beleid naar het andere verhuizen.

Als de drie types van politieke cultuur voorgesteld in de bovenstaande tabel de zuivere vorm van politieke cultuur zijn, kunnen we drie types van systematisch gemixte politieke culturen onderscheiden:

The Parochial-Subject Culture. Dit is een type van politieke cultuur waarin een substantiële proportie van de populatie de exclusieve claims van de diffuse stammen-, dorps-, of feodale autoriteit hebben verworpen en een zekere trouw hebben ontwikkeld tegenover een meer complex politiek systeem met gespecialiseerde centrale overheidsstructuren. De kronieken en de geschiedenis van de meeste naties bevatten deze vroege fase van overgang van lokaal parochialisme naar centrale autoriteit. Maar de overgang kan stabiliseren op een ogenblik dat er nog geen volledige subject cultuur is ontwikkeld. De overgang van een parochiaal naar een subject politieke cultuur kan op verschillende tijdstippen stabiliseren en verschillende politieke, psychologische en culturele mixen voortbrengen.

The Subject-Participant Culture. De manier waarop de overgang van parochiaal naar subject cultuur is verlopen, heeft een groot effect op de manier waarop de overgang van subject naar participant cultuur verloopt. In de overgang van een subject naar een participant cultuur kunnen de parochiale en lokale autonomieën, wanneer ze overleven, bijdragen tot de ontwikkeling van een democratische infrastructuur. In de “mixed subject-participant culture” heeft een substantieel deel van de bevolking gespecialiseerde input oriëntaties en actieve set van zelforiëntaties verworven, terwijl de rest van de bevolking georiënteerd blijven ten aanzien van een autoritaire overheidsstructuur en een relatieve passieve set van zelforiëntaties hebben.

The Parochial-Participant Culture. In de parochial-participant culture zien we de huidige problemen van culturele ontwikkeling in vele opkomende staten (alweer we schrijven 1963!). In de meeste van deze landen is de politieke cultuur dominerend parochiaal. De structurele normen die werden geïntroduceerd zijn gewoonlijk participant. Maar er is geen structuur om op te steunen (geen democratische maar ook geen autocratische), noch een bureaucratie die steunt op loyale subjecten, noch een infrastructuur die voortkomt uit verantwoordelijke en competente burgers. Het probleem (of de uitdaging) is het parochiale systeem te doordringen zonder de output zijde ervan te vernietigen, en om hen om te vormen tot belangengroepen aan de input zijde.

Politieke subcultuurZelfs de meest ontwikkelde participant culture zal overblijvende groepen van subjects en parochials bevatten. En zelfs binnen het deel van de cultuur dat georiënteerd is naar participatie zullen er significante en persisterende verschillen van politieke oriëntaties zijn. De auteurs

6

Page 7: Facultair Overlegorgaan Sociale Wetenschappen … · Web viewSolidariteit op de werkvloer maakt deel uit van een ruimere sociale solidariteit 2) Kerk en religiositeit: Georganiseerde

DAAN DELESPAUL 2017-2018

gebruiken de term “subcultuur” om te verwijzen naar deze onderdelen van politieke culturen. Ze onderscheiden twee types van subculturele breuklijnen.

Ten eerste verwijst de term naar groepen van de populatie die op een bepaalde manier georiënteerd zijn ten opzichte van de beleidsinput en output (voorbeeld liberalen, socialisten), maar wel trouw zijn aan de politieke structuur (wat wij ook ideologische breuklijnen noemen).

Maar de breuklijnen die voorkomen in gemixte systemen zijn de soort breuklijnen waar we het meeste in geïnteresseerd zijn. In een mixed parochial-subject culture zou een deel van de populatie georiënteerd zijn naar diffuse traditionele autoriteiten, en een ander deel naar de gespecialiseerde structuur van het centrale autoritair systeem. Deze politieke cultuur kan gekarakteriseerd worden door zowel verticale als horizontale breuklijnen. Een succesvolle overgang van een subject naar een participant cultuur houdt de verspreiding van positieve oriëntaties tegenover de democratische structuur, de aanvaarding van de burgerplichtnormen, en de ontwikkeling van een gevoel van burgercompetentie onder een substantieel deel van de bevolking in. Deze oriëntaties kunnen gecombineerd worden met subject en parochiale oriëntaties of ze kunnen er mee conflicteren.

De civic culture: een gemengde politieke cultuurDe civic culture is meer dan alleen de politieke cultuur die beschreven wordt in civic textbooks met de nadruk op rationele participatie binnen de input structuren van de politiek. De civic culture deelt veel met dit rationeel actief model, maar het is in feite zo’n cultuur plus iets meer.

De burgerlijke cultuur is, in de eerste plaats, een trouwe participatie cultuur. Individuen zijn niet alleen georiënteerd naar de politieke input maar ze zijn ook positief georiënteerd naar de input structuren en het input proces. De civic culture is een participant political culture waar de politieke cultuur en de politieke structuur congruent zijn. Individuen worden deelnemers (participanten) aan het politieke proces, maar ze moeten hun oriëntaties als subject of parochial niet opgeven. De subject en parochiale oriëntaties zijn congruent met de participant politieke oriëntaties. De meer traditionele politieke oriëntaties begrenzen het individuele engagement tot de politiek en maken het engagement milder. In die zin, “managen” de subject en parochiale oriëntaties of houden ze de participant oriëntaties op hun plaats.

Micro- en macropolitiek: politieke cultuur als de “connecting link”Micropolitiek = de focus op het individu, zijn politieke attitudes en motivaties, zowel als individu als een lid van een staal van een grotere populatie.

Macropolitiek = de studie van politiek met de structuren en functies van politieke systemen, instituties, en organisaties en hun effecten op het openbare beleid.

De auteurs suggereren dat de relatie tussen attitudes en motivaties van de individuen die politieke systemen uitvinden, en het karakter en de uitvoering van het politieke systeem systematisch kan worden ontdekt door het concept van de politieke cultuur dat we hierboven hebben geschetst. Met andere woorden de “connecting link” tussen micro- en macropolitiek is politieke cultuur. Elk beleid kan beschreven en vergeleken worden in termen van (1) zijn structurele-functionele karakteristieken, en (2) zijn culturele, subculturele en rol-culturele karakteristieken. Onze analyse van types van politieke cultuur is een eerste poging om het fenomeen van de individuele politieke oriëntatie aan te pakken op zo een manier dat ze systematisch gerelateerd wordt met het fenomeen van politieke structuur.

Het is de hypothese van de auteurs dat het onderzoek van het belang van het specifiek leren van oriëntaties tegenover politiek en ervaring met het politieke systeem lange tijd ernstig werd onderbenadrukt. Dit leren is niet alleen cognitief van karakter, maar betrekt ook politieke gevoelens, verwachtingen en evaluaties die vooral voortkomen uit politieke ervaringen en die meer zijn dan de simpele projectie van basisbehoeften en attitudes die het product zijn van socialisatie tijdens de kindertijd.

Startend vanuit de oriëntatie en psychologische voorwaarden van verschillende types van politieke structuur, zijn we in een betere positie om hypotheses te formuleren over de categorieën van persoonlijkheden en socialisatie praktijken die waarschijnlijk congruente politieke culturen en stabiel beleid voortbrengen. In het geval van de civic culture, kunnen we zeggen dat het patroon van socialisatie het individu mogelijk maakt de onvermijdelijke dissonantie tussen zijn diffuse primaire, zijn gehoorzame output en actieve input rol te managen.

7

Page 8: Facultair Overlegorgaan Sociale Wetenschappen … · Web viewSolidariteit op de werkvloer maakt deel uit van een ruimere sociale solidariteit 2) Kerk en religiositeit: Georganiseerde

DAAN DELESPAUL 2017-2018

4. Robert D. Putnam (1993): Making Democracy Work-Civic Traditions in Modern Italy (83-120)

Een onderzoek naar de performantie van Italiaanse regionale overheden. Een verschil in cultuur verklaart de verschillen aan outputkant.

InleidingDe aanleiding van het onderzoek is het noord-zuid verschil tussen Italiaanse regio’s: de regionale overheden in het noorden presteren beter, halen een hoger democratisch gehalte dan de zuidelijke regio’s. Dit kan niet aan de instituten liggen, die zijn immers overal dezelfde. Anderzijds kan het aan de economie liggen, maar dat is geen volledige verklaring: cultuur?

Wat bepaalt de prestatie, het succes van de publieke instituten?

Focus op 2 mogelijkheden:

1. “socio-economic modernity” d. i. de sociale en economische ontwikkeling, de industriële revolutie

2. “civic community” d. i. patronen van burgerbetrokkenheid en sociale solidariteit

Socio-economische moderniteit:Er blijkt een verband tussen welvaart, economische ontwikkeling en goede bestuurlijke prestaties, maar het is beperkt, het gaat niet altijd op. We kunnen echter niet tonen wat dit verband precies inhoudt (moderniteit als een oorzaak of een gevolg van overheidsoptreden), het lijkt ook dat er een derde factor is die beiden beïnvloedt, een schijnverband. We kunnen dus niet stellen dat economie de enige factor is of überhaupt causaliteit verklaart.

De “civic community” (burgergemeenschap): theoretische speculatiesSucces van overheid?

Republican school (o.a. Machiavelli): het goed functioneren van de overheid hangt af van het karakter van haar burgers, “civic virtue” (burgerdeugd). Nadruk op de gemeenschap.

Liberale school: niet uitgaan van een deugdzame, op het publieke betrokken burger, maar individualisme van burgers. Overheid moet de democratie vrijwaren van de niet-deugdzamen.

Revisionist wave (nieuwe republikeinse theoretici): theorie in verband met effectief democratisch bestuur. De focus ligt bij de gemeenschap, deze civic community faciliteert het functioneren van de democratie: “als de proportie van niet-deugdzame burgers toeneemt zullen liberale samenlevingen minder goed kunnen functioneren.”

Onderzoek Putnam:Is het succes van een democratische overheid afhankelijk van de mate waarin zij het ideaal van een “civic community” benaderen?

Vooraf: wat houdt “civic community“ in (in praktijk)?

Civic engagement : actieve participatie in publieke zaken, belangstelling en inzet voor publieke zaken.

”Self interest properly understood ”: je eigenbelang plaatsen in een ruimer kader van publieke noden en gemeenschappelijke baten.

Politieke gelijkheid : gelijke rechten en plichten voor iedereen, reciprociteit (wederzijdse gelijke behandeling) en zelfbestuur in plaats van autoriteit en afhankelijkheid.

Solidariteit, vertrouwen en tolerantie: hulpvaardig, respectvol en vertrouwensvol tegenover de andere. Vooral wederzijds vertrouwen draagt bij tot “civic community” kans op opportunisme daalt.

Verenigingen : sociale structuren van coöperatie. Associaties van burgers dragen door hun interne en externe effecten bij aan de stabiliteit van democratisch bestuur: intern: door deelname aanleren van vaardigheden van samenwerken en een gevoel van gedeelde verantwoordelijkheid. Extern; ”intrest articulation”, het vormen van een duidelijk standpunt en “interest aggregation”, verschillende meningen verenigen en naar één doel leiden.

8

Page 9: Facultair Overlegorgaan Sociale Wetenschappen … · Web viewSolidariteit op de werkvloer maakt deel uit van een ruimere sociale solidariteit 2) Kerk en religiositeit: Georganiseerde

DAAN DELESPAUL 2017-2018

Het testen van de theorie:Operationaliseren van civicness om de vraag te kunnen onderzoeken of er een verband bestaat tussen de civicness van een gemeenschap en de kwaliteit van het bestuur:

Vier indicatoren

1. Lidmaatschap van verenigingen: In Italië als geheel hebben sportverenigingen de meeste leden- in dit domein is er

algemeen de meest activiteit Er bestaan verschillen in aantal lidmaatschappen tussen de regio’s

2. Het lezen van de krant (=politieke kennis): Beter geïnformeerde burger, beter kunnen deelnemen aan publiek overleg krant lezen

wijst op interesse voor publieke zaken Opnieuw grote variatie tussen de verschillende regio’s

3. Opkomst van de kiezers, meer bepaald opkomst bij nationale referenda Gaat over belangrijke publieke zaken afwezigheid van persoonlijke, van weinig

burgerzin getuigende motieven Sterke en stabiele regionale verschillen rond de verschillende onderwerpen

4. Het uitbrengen van voorkeurstemmen Indicator voor de afwezigheid van civic community, want duidt (in Italië) op

personalisme, cliëntilisme Stabiele regionale verschillen Negatieve correlatie tussen opkomst bij referenda en het uitbrengen van

voorkeurstemmen (figuur 4.3, p.95)) er is weinig verschil in hoeveelheid participatie, wel in de kwaliteit ervan. Wie in Calabrië bijvoorbeeld gaat stemmen doet dit eerder op basis van persoonlijke (cliëntelistische) overwegingen dan omwille van politieke visie.

Deze 4 indicatoren correleren onderling sterk: wie de krant leest is meer geneigd ook in een vereniging te zitten.

Combinatie van de indicatoren in één Civic Community IndexFiguur 4.4: niveau van civic-ness van de Italiaanse regio’s in kaart gebracht. (p.97)

Figuur 4.5: hoe meer een regio ‘civic’ is hoe effectiever haar bestuur, deze relatie is sterker dan die tussen economische ontwikkeling en prestatie van de instituties (p.98): cultuur als basis economische prestaties.

Sociale en politieke implicates van ‘civic-ness’1. Gedrag van de burgers:Bewijzen dat in samenlevingen die minder ‘civic’ zijn, politiek engagement méér wordt afgedwongen via personalistische, patroon-cliënt netwerken:

Zelfbeschrijvingen van regionale politici correleren sterk met de Civic Community Index (als de politici het politieke leven als ‘clientilistic’ omschreef was er een lage score op de index)

“particularized contacting” burgers van ‘niet-civic communities’ contacteren hun vertegenwoordigers méér, maar dit is vaak over patronage (bv. helpen aan een job) terwijl burgers van ‘civic communities’ dit minder doen en vooral vragen omtrent het beleid stellen.

2. Karakter van de politieke elites De regionale elite van gemeenschappen die weinig ‘civic’ zijn komen uit het meer

geprivileerde deel (hoog opgeleid) van de bevolking. Een belangrijk deel van de politieke leiders van meer ‘civic’ gemeenschappen komen uit meer bescheiden achtergronden. Reden? Cliëntelisme verticale relaties van autoriteit en afhankelijk en dus elitaire politiek

Politieke leiders van regio’s met een hogere mate van burgerzin staan duidelijk positiever tegenover politieke gelijkheid dan die met een mindere mate. ’Index of Support for Political Equality’ scores van de regionale leiders weerspiegelen de mate van civic community van die regio (figuur 4.8)

9

Page 10: Facultair Overlegorgaan Sociale Wetenschappen … · Web viewSolidariteit op de werkvloer maakt deel uit van een ruimere sociale solidariteit 2) Kerk en religiositeit: Georganiseerde

DAAN DELESPAUL 2017-2018

Één indicator van ‘civic community’ is de mate van voorkomen van lokale culturele en recreatieve associaties (zie boven).De ‘civic’ context heeft echter een verschillend effect op verschillende soorten van organisaties namelijk de vakbonden, de kerk en de politieke partijen.1) Vakbonden:

Lidmaatschap in Italië is vrijwillig Ideologische fragmentering van de arbeidersbeweging je kan een keuze maken Politieke motivatie en ideologische traditie is een beter voorspeller van het lidmaatschap

dan de economische structuur In meer ‘civic’ regions hebben de vakbonden meer leden (het dubbel) Solidariteit op de werkvloer maakt deel uit van een ruimere sociale solidariteit

2) Kerk en religiositeit: Georganiseerde religie is een alternatief voor de burgergemeenschap Manifestaties van religiositeit en clericalisme (zoals kerkgang) zijn negatief gecorreleerd

met engagement van de burger (zoals krant lezen, politieke discussies houden): religieus en politiek actief zijn in niet cumuleerbaar

Na wereldoorlog II: De Katholieke Actie: katholieke lekenorganisaties – netwerk van culturele, recreatieve en educatieve activiteiten participatieve kant katholicisme maar enkel in het noorden populair, stort in na Tweede Vaticaans Concilie.

Na 1960: civic community wordt een seculiere gemeenschap

3) Politieke Partijen: Lidmaatschap van politieke partijen is ongeveer gelijk voor de meest ‘civic’ regio als voor

de minste, de burgers in de laatste zijn niet minder partijganger of politiek actief Partijlidmaatschap heeft wel een verschillend karakter, in de regio’s die minder burgerzin

kennen is de partij drager van het cliëntelisme. Lidmaatschap fungeert als ‘connectie’ met de invloedrijken.

Civic attitudes Burgers van minder ‘civic’ regio’s voelen zich meer uitgebuit, vervreemd en machteloos.

De verschillen in machteloosheid die er bestaan tussen de hoger en lager opgeleiden zijn groter in minder ‘civic’ regio’s.

In een gemeenschap met een sterke burgerdeugd, die deugden als eerlijkheid, vertrouwen en trouw aan de wet inhoudt, kan men minder aan ‘free-riding’ doen. Vrijheid is hier het deelnemen aan en handelen uit gemeenschappelijke beslissingen. Je verwacht dat de ander zich ook aan de regels houdt. In samenlevingen die in mindere mate een burgergemeenschap zijn is er minder vertrouwen, je verwacht dat de andere vals speelt, je wil niet ‘cuckolded’ (de bedrogen echtgenoot) zijn, als enige naïef de regels volgen.

Er ís daadwerkelijk meer corruptie in de minder ‘civic’ regio’s van Italië.

In de ‘niet-civic communities’ is er vraag naar hiërarchie, orde herstelling door hogere krachten: de politie. Maar, op de plaatsen waar een sterker bestuur wordt gevraagd kan het bestuur, als zij democratisch wil blijven, niet sterker zijn, door de afwezigheid van wil tot samenwerken en er minder zelf-handhaving is.

‘civic-ness’ als predictor van levensgenoegen. Burgers van ‘civic communities’ zeggen meer tevreden met het leven te zijn.

Veel theoretici associëren ‘civic communities’ met traditionele (kleine, hechte, pre-moderne) samenlevingen dus als een wereld die niet meer bestaat. Deze studie brengt echter naar voren dat de gebieden met een gebrek aan burgerzin net de traditionele zuidelijke steden zijn: de meer moderne regio’s concentreren netwerken van sociale solidariteit met toewijding van haar burgers voor de publieke belangen. Modernisatie hoeft niet de ondergang van de ‘civic commmunity’ aan te kondigen. Het bewijsmateriaal verzameld in Italië is ondubbelzinig:

De ‘civic context’ (aanwezigheid van burgerzin) heeft betekenis voor de manier waarop instituties werken

De mate waarin het sociale en politieke leven van een regio het ideaal van een ‘civic community’ benaderen is de belangrijkste factor in het verklaren van ‘good governance’

10

Page 11: Facultair Overlegorgaan Sociale Wetenschappen … · Web viewSolidariteit op de werkvloer maakt deel uit van een ruimere sociale solidariteit 2) Kerk en religiositeit: Georganiseerde

DAAN DELESPAUL 2017-2018

5. Robert D. Putnam (2000): Bowling Alone. The Collapse and Revival of American Community (p.

31-47,134-147,277-284)De participatie en het vertrouwen in de politiek neemt jaar na jaar af. Dit gegeven hangt nauw

samen met het begrip ‘sociaal vertrouwen’, het algemeen vertrouwen in de ander dat intergenerationeel daalt. Het social vertrouwen als fundament van social kapitaal is nu vervangen

door formele instituties zoals het recht.

Chapter 2: Politieke Participatie. (p. 31-47)

Waarom gaat de kiezersopkomst bij verkiezingen steeds achteruit in Amerika?De politieke betrokkenheid van de Amerikanen is de laatste 30 jaar veel veranderd. In vergelijking met andere democratische staten scoort de VSA goed in politieke participatie. Er zijn verschillende manieren om de rechten van de burger te doen gelden. Het grote probleem is dat steeds minder Amerikanen gebruik maken van hun stemrecht. In 1960 ging nog 62.8% naar de stembus, terwijl dit nog maar 48.9% was in 1996. Er zijn twee mogelijke verklaringen voor dit fenomeen. Ten eerste denkt men aan de problemen rond registratie. Maar hier kan men direct tegen inbrengen dat de registratie-eisen de laatste 40 jaar sterk versoepeld zijn. Ten tweede denkt men aan de vele zwarten in het Zuiden van wie het stemrecht werd ontnomen om verscheidene redenen, check figuur p.32. De civil rights movement heeft hier in de jaren ’60 echter een eind aangemaakt. De kiezersopkomst na de jaren ’60 van de zwarten heeft zelfs de steeds lager wordende opkomst van de andere kiezers gemaskeerd.

Men kan zich de vraag stellen wie deze niet-stemmers zijn en waarom ze niet willen stemmen? Hier zijn vele verklaringen voor te vinden, maar de afname van de kiesopkomst is te wijten aan het verschuiven van de oude naar een nieuwe generatie.

Wat is nu het verband tussen sociale verandering en generationele verandering? Het heeft te maken met twee heel verschillende processen. Ten eerste bestaat er een intracohort type van verandering. Dit type van sociale verandering gaat over een gelijktijdige verandering in één richting in smaak en gewoontes overheen alle generaties. Dit type treedt heel snel op en kan snel weer veranderen, denk maar aan mode. Ten tweede hebben we het over een intercohort type van verandering. Dit type van sociale verandering treedt veel trager op en is moeilijk om terug om te keren. Het gaat meer over een generatie verschil. Elke generatie heeft zijn morele waarden en die kunnen nogal verschillen van generatie tot generatie, denk maar aan Woodstock. De meeste sociale veranderingen zijn zowel intercohort als intracohort. Maar indien we het over kiezersopkomst hebben, zien we dat dit eerder een intercohort type van sociale verandering is. De generatie van de babyboomers, onze ouders, en de generatie erna, wij, maken een groot deel uit van het kiezerskorps en gaan steeds minder stemmen.

Stemmen is de normaalste vorm van politieke activiteit en gaat over de meest fundamentele vorm van gelijkheid. We zien dat stemmen samengaat met actief zijn in de gemeenschap. Dus wanneer het slecht gaat met de kiezersopkomst, gaat het ook slecht met de gemeenschap, hieruit kunnen we besluiten dat deze afname alarmerend is voor Amerika.

Politieke interesse in openbare diensten gaat sterk achteruit bij de jongere generatie.De politieke kennis tussen de Amerikanen nu en die van vijftig jaar geleden zijn ongeveer hetzelfde. Aan de ander kant is het niveau van onderwijs en informatie nu wel veel hoger dan toen maar de kennis is niet mee geëvolueerd. Op 25 jaar tijd is de interesse in openbare diensten en het nieuws sterk achteruitgegaan.

Het meest onrustbarend is het verschil in politieke kennis bij verschillende generaties. Dit weerlegt de stelling dat de politiek saaier zou zijn geworden (het is geen natie probleem, maar generatie probleem). De postbabyboomgeneratie interesseert zich simpelweg veel minder in openbare diensten. De jeugd van vandaag weet minder over politiek dan hun ouders en minder dan de jeugd van 20 jaar geleden op dat moment over politiek wist.

Financieel kapitaal vervangt sociaal kapitaal. Het probleem van de generatiekloof dreigt te leiden tot een groeiende afname in politieke participatie in de toekomst. Gaan stemmen en politieke interesse zijn geen vormen van sociaal

11

Page 12: Facultair Overlegorgaan Sociale Wetenschappen … · Web viewSolidariteit op de werkvloer maakt deel uit van een ruimere sociale solidariteit 2) Kerk en religiositeit: Georganiseerde

DAAN DELESPAUL 2017-2018

kapitaal, vermits het individuele activiteiten zijn. Indien we het van de positieve kant bekijken kunnen we stellen dat de partijorganisaties sterker zijn dan ooit tevoren. Ze zijn rijker, groter en professioneler. Zoals we zien in de figuur, is er nog nooit zoveel betaald personeel geweest. De afname van de kiezersopkomst en de politieke interesse staat haaks op de vooruitgang van de partijorganisaties. Dit is een vreemde parodie. Verder zien we dat ook de partij-identificatie sterk achteruitgaat, het fenomeen van de zwevende kiezer. De andere figuur toont aan dat de medewerking aan verkiezingen van de burger eveneens sterk achteruitgaat. Men stelt zich de vraag hoe het mogelijk is dat de partijorganisaties zo gezond zijn terwijl de burgers steeds minder meehelpen? Het antwoord is duidelijk. De toename in professionalisering en commercialisatie maken het voor de partijen mogelijk veel mensen te contacteren zonder veel helpers nodig te hebben. Zo kunnen we besluiten dat het financieel kapitaal het sociaal kapitaal heeft vervangen. Men kan zich hierbij de vraag stellen of deze verandering niet volledig voorbij gaat aan het nut van politieke participatie.

Politieke participatie op het lokale niveau.Wanneer we kijken naar de politieke participatie op lokaal niveau, kunnen we stellen dat die niet geruststellender is dan op nationaal niveau. Onderzoek heeft geleid tot gegevens over burgeractiviteiten, gaande van het tekenen van petities tot het zich kandidaat stellen voor een verkiezing. De resultaten zijn bedroevend. Men stelt zich de vraag hoe het mogelijk is dat dit in 30 jaar tijd zo achteruit is gegaan. De eerste figuur toont aan hoe de betrokkenheid van de burger in partij- en campagneactiviteiten achteruit is gegaan. De cijfers tonen aan dat de betrokkenheid van de burger nog kleiner is dan de kiezersopkomst. Vooral de cijfers van kandidaten die voor een partij opkomen zijn sterk gedaald.

Hoe staat het met gemeentelijke activiteiten? De tweede figuur toont aan dat ook hier de participatie sterk gedaald is. De derde figuur gaat over de trends in verschillende vormen van publieke expressie. Ook hier wordt een sterke daling gemeten. Hier kan men echter wel een verschil meten in de verschillende vormen. Indien we ze opdelen in participatie die of individueel kunnen gebeuren of in een groep, zien we dat de individuele participatie, vb brief schrijven naar congres, minder sterk is gedaald dan de participatie in groep, zie tabel. Indien we pogen een verschil te maken op basis van streek, geslacht, populatiegroepen, zien we dat de daling van participatie alom tegenwoordig is.

Besluit.Als we de som maken, kunnen we stellen dat de participatie sterk gedaald is. We vinden een daling van participatie in het zich kandidaat stellen en in een brief schrijven naar het congres of naar een krant. We merken een daling op in politieke interesse, publieke zaken, gaan stemmen, naar publieke bijeenkomsten gaan, betrokkenheid in politieke partijen en in burgerorganisaties.

Kunnen we dit allemaal uitleggen aan de hand van aliënatie van de politiek en een daling van vertrouwen in politieke activiteit? We zien dat er een sterke daling is in het vertrouwen dat de burger in zijn president of regering stelt. Misschien is dit de verklaring. Misschien is de disfunctionele lelijkheid van hedendaagse politiek en de afwezigheid van grote, collectieve projecten de reden waarom we onze energie weg hebben geleid van conventionele politiek naar meer effectieve kanalen.

Chapter 8: Reciprocity, Honesty and Trust (p. 134-147)Opmerking vooraf: ‘sociaal’ kan hier volgens mij best als ‘maatschappelijk’ geïnterpreteerd worden.

‘Generalized reciprocity’ De toetssteen van sociaal kapitaal is het principe van ‘generalized reciprocity’: Ik doe dit voor jou,

zonder onmiddellijk iets in de plaats te verwachten en misschien zelfs zonder jou te kennen, wetende dat jij of iemand anders op een dag iets voor mij zal doen. (Filosoof Michael Taylor

spreekt in deze context van ‘korte–termijn altruïsme’ en ‘lange-termijn eigenbelang’.)

Deze norm van ‘generalized reciprocity’ is zo fundamenteel voor onze beschaving (Amerikaanse nvdr), dat alle belangrijke ethische codes die regel op de een of andere manier bevatten. Het is belangrijk in te zien dat leden van een samenleving waarin dit principe van reciprociteit ingebakken zit, vinden dat op die manier hun eigenbelang het best gediend wordt. Volgens Alexis de Tocqueville (begin 19eE) werkte de Amerikaanse democratie niet omdat de Amerikanen een of andere onmogelijke idealistische regel van onbaatzuchtigheid (selflessness) volgden, maar wel omdat ze geloofden dat door zo te handelen hun eigenbelang het best gediend werd.

12

Page 13: Facultair Overlegorgaan Sociale Wetenschappen … · Web viewSolidariteit op de werkvloer maakt deel uit van een ruimere sociale solidariteit 2) Kerk en religiositeit: Georganiseerde

DAAN DELESPAUL 2017-2018

Een maatschappij gebaseerd op ‘generalized reciprocity’ is efficiënter dan een wantrouwige maatschappij, om dezelfde reden als geld meer efficiënt is dan ruilhandel: door lagere transactiekosten. Eerlijkheid en vertrouwen overstijgen de onvermijdelijke wrijvingen die eigen zijn aan het sociaal leven.

Eerlijkheid en Vertrouwen‘Eerlijk duurt het langst’, maar enkel indien anderen daar net zo over denken. Sociaal vertrouwen

is een waardevol gemeenschapsgoed als en slechts als het gegarandeerd wordt. We zijn beiden beter gediend met oprecht te zijn tegenover elkaar dan wanneer we zouden afzien van samen te

werken, omdat we vrezen verraden te worden. Betrouwbaarheid en niet simpelweg ‘vertrouwen’ is de sleutel.

De vraag is nu welke garantie we hebben dat anderen te vertrouwen zijn?

a) Een juridisch systeem (rechtbanken, wetten…); probleem hierbij zijn de hoge transactiekosten. Volgens Diego Gambetta zijn dergelijke samenlevingen ook minder efficiënt, kostelijker en onaangenamer.

b) Sociale netwerken waarin onze dagelijkse transacties ingebed zijn; mensen zullen mogelijk in de toekomst nog met elkaar in contact komen, waardoor reputatie een belangrijkere rol zal spelen dan de verleiding om in een bepaalde situatie te bedriegen. Dus: eerlijkheid wordt aangemoedigd door hechte sociale netwerken.

Er bestaan twee soorten vertrouwen, afhankelijk van eerlijkheid gebaseerd op persoonlijke ervaring of eerlijkheid gebaseerd op een algemene gemeenschapsnorm. Men spreekt van ‘thick

trust’ wanneer dit vertrouwen ingebed is in persoonlijke relaties die sterk, vaak voorkomend en in bredere netwerken gesitueerd zijn. ‘Thin trust’, vertrouwen in ‘the generalized other’ is eerder gebaseerd op de achtergrond van gedeelde sociale netwerken en ervaringen van reciprociteit.

Men zou kunnen zeggen dat ‘thin trust’ meer bruikbaar is dan ‘thick trust’, omdat dit verder strekt dan de mensen die we persoonlijk kunnen kennen. Wanneer het sociale weefsel van een

gemeenschap echter verzwakt, vermindert ook haar effectiviteit wat betreft het overdragen en ondersteunen van reputaties en verzwakt ook haar kracht om normen als eerlijkheid, ‘generalized

reciprocity’ en ‘thin trust’ te onderbouwen.

De politieke wetenschappers Wendy Rahn en John Transue zien het concept van sociaal vertrouwen (cfr. ‘thin trust) als het geven van het voordeel van de twijfel aan de meeste mensen.

In die zin is sociaal vertrouwen heel nauw verbonden met vele andere vormen van burgerlijk engagement en sociaal kapitaal. Mensen die hun naasten meer vertrouwen, blijken aldus ook meer vrijwilligerswerk te verrichten, geven meer aan goede doelen, participeren meer in de politiek en

in het verenigingsleven… In deze context hebben psychologen aangetoond dat mensen die geloven in de eerlijkheid van anderen, zelf minder gemakkelijk liegen, bedriegen, stelen en meer respect

tonen voor de rechten van anderen.

Uit bovenstaande blijkt het belang van een groot onderzoek naar trends in sociaal kapitaal de laatste tientallen jaren in Amerika om te achterhalen hoe reciprociteit en sociaal vertrouwen geëvolueerd zijn (niet zozeer ‘thick trust’ maar veeleer ‘thin trust’ in de anonieme medemens). De centrale vraag in dit hoofdstuk is dus:

Hoe zijn de trends in sociaal kapitaal en burgerlijk engagement die we tot dusver hebben ontdekt, terug te vinden in trends van eerlijkheid en sociaal vertrouwen in Amerika?Opmerking: Sociaal vertrouwen (‘social trust’) is niet gelijk aan vertrouwen in de overheid of in maatschappelijke instituties, hoewel beide mogelijk wel gecorreleerd zijn met elkaar.

Om de centrale vraag te beantwoorden, kan er gelukkig een beroep gedaan worden op gestandaardiseerde vragenlijsten over sociaal vertrouwen en eerlijkheid, die reeds tientallen jaren bij Amerikanen afgenomen werden. Spijtig genoeg zijn de antwoorden ambigu. De meest voorkomende onderzoeksvraag bijvoorbeeld; “Algemeen gezien, zou u zeggen dat de meeste mensen te vertrouwen zijn, of dat men niet voorzichtig genoeg kan zijn in de omgang met anderen?” Dat het antwoord ‘de meeste mensen zijn te vertrouwen’ heden ten dage minder frequent voorkomt, kan op verschillende manieren geïnterpreteerd worden;

1) mensen zijn minder eerlijk geworden; 2) het gedrag van de anderen is niet veranderd, maar we zijn meer wantrouwig geworden; 3) gedrag niet veranderd, maar we krijgen nu meer informatie over bedrog, mogelijk door

scherpere mediaberichtgeving.Conclusie: antwoord niet eenvoudig!

13

Page 14: Facultair Overlegorgaan Sociale Wetenschappen … · Web viewSolidariteit op de werkvloer maakt deel uit van een ruimere sociale solidariteit 2) Kerk en religiositeit: Georganiseerde

DAAN DELESPAUL 2017-2018

Merk het belang op van eigen sociale ervaringen. In het algemeen hebben armen bijvoorbeeld minder vertrouwen dan rijken, mogelijk omdat rijken door anderen met meer respect en

eerlijkheid worden behandeld. Het is dus aannemelijk te veronderstellen dat de eerder beschreven sociale patronen eigen ervaringen weerspiegelen, eerder dan verschillende psychische neigingen

tot wantrouwen.

Vb.: De grootte van een stad. Praktisch alle vormen van altruïsme (vrijwilligerswerk, gemeenschapsprojecten, …) komen meer voor in kleine steden. Criminaliteit is lager, …

Samengevat: het grotere wantrouwen tegenover de ‘generalized other’ bij inwoners van grote steden, is niet een of andere specifieke paranoia die voortkomt uit het stadsleven, maar is een realistische weerspiegeling van eigen concrete ervaringen en sociale normen uit hun omgeving. Steden zijn misschien wel ‘vrijere’ plaatsen om te leven, maar de prijs die men betaalt is een verzwakte ‘thin trust’!

Opmerking: Sociaal wantrouwen is niet zuiver objectief. Persoonlijk cynisme, paranoia, projecties van eigen neigingen tot oneerlijkheid, … spelen ook een zekere rol. Men kan veronderstellen dat iemands visie tegenover basisbegrippen als ‘reciprocity’ en ‘generalized trust’ specifiek worden beïnvloed op jonge leeftijd door persoonlijke ervaringen en sociale gewoonten. Men spreekt hier van de ‘vormende’ jaren.

De meeste Amerikanen geloven dat we in een minder betrouwbare samenleving leven dan onze ouders (zie fig. 37, p. 139), een trend die door de jaren heen enkel verduidelijkte. Maar misschien is dit enkel een bewijs dat nostalgie, het verlangen naar het verleden, populairder is geworden?

Om een duidelijkere uitspraak te doen kan men onze gevoelens van vandaag vergelijken met wat voorgaande generaties antwoordden op identieke vragen (cfr. Figure 38, p. 140), i.p.v. te vergelijken met hoe wij ons voorstellen dat een vorige generatie zich zou gevoeld hebben. Hieruit blijkt dat sociaal vertrouwen steeg van midden ’40 tot midden ‘60, met een piek in ’64 (zoals ook andere indicatoren van sociaal kapitaal deden). Vanaf midden ’60 zien we een lange termijn dalende trend van sociaal vertrouwen. Vooral bij jongeren zien we een sterke daling, vooral vanaf 1985.

De grootste verklarende voor deze daling kan toegeschreven worden aan de generationele opvolging (‘generational succession’, cfr. Figure 39, p. 141). Op het einde van de 20e E, zien we dat een generatie met een vertrouwenscoëfficiënt van rond de 80% (mensen geboren vóór 1930), snel vervangen werd door een generatie (geboren na 1960) met een vertrouwenscoëfficiënt die bijna de helft daarvan bedroeg. Het resultaat is een sterke daling van het algemeen sociaal vertrouwen, hoewel we zien dat elke individuele cohorte nagenoeg even vertouwend bleef als vroeger.

Eerder werd de assumptie aangehaald dat uitingen van sociaal vertrouwen in de eerste plaats weerspiegelingen zijn van persoonlijke ervaring, vooral op jongere leeftijd. Het sociaal wantrouwen onder jongeren moet echter eerder gezien worden als een weerspiegeling van de maatschappelijke waarden van de laatste decennia. Jongeren vermelden dat zij ervaren dat de meeste mensen echt niet betrouwbaar zijn. Misschien is de ‘thick trust’ (vertrouwen in persoonlijke vrienden) wel even sterk als vroeger, ‘thin trust’ wordt schaarser.

Een andere indicator van de dalende ‘generalized trust’ en ‘reciprocity’, is de verminderde respons op opinieonderzoek (meer dan gehalveerd sinds de ’60). Volgens recent onderzoek hiernaar zou de sociale desintegratie (‘social disconnectedness’) deze trend gedeeltelijk verklaren. Opm.: trend niet terug te vinden in onderzoek per post, enkel in face-to-face en telefonische interviews; kan erop kan wijzen dat de lagere respons te wijten is aan de bedreiging die uitgaat van persoonlijk contact met anonieme vreemden.

(Ongerustheid kan ook helpen verklaren waarom de proportie niet-vermelde telefoonnummers met 2/3 toenam de laatste 2 decennia en waarom telefoon-screening meer dan verdrievoudigde in de jaren ’90 (vooral bij jongeren). Technologische ontwikkelingen kunnen hiertoe bijgedragen hebben (oorzaak), hoewel deze technologieën zelf een marktvraag beantwoorden (gevolg van ongerustheid)

Andere indicator van de verlaagde reciprociteit (nauw verbonden hiermee ook burgerschap);

Daling vrijwillige respons op volkstellingenformulieren per post met meer dan ¼ tussen 1960 en 1990 (vooral jongeren, Afro-amerikanen,… de groepen met laagste sociaal vertrouwen; Opm.: aliënatie lijkt hier geen rol te spelen.

Als fair-play tegenover de ‘generalized other’ minder algemeen is vandaag, zou dat zich moeten manifesteren in interacties met vreemden. Als indicator kan hier ‘rijgedrag’ gebruikt worden. Volgens een studie van de ‘American Automobile Association foundation for Traffic Safety’, steeg hevig aggressief rijgedrag (‘road rage’) met meer dan 50% tussen 1990 en 1996 (illustratie:

14

Page 15: Facultair Overlegorgaan Sociale Wetenschappen … · Web viewSolidariteit op de werkvloer maakt deel uit van een ruimere sociale solidariteit 2) Kerk en religiositeit: Georganiseerde

DAAN DELESPAUL 2017-2018

houding tgo te snel rijden versoepelde laatste jaren). We weten allemaal dat chauffeurs tegenwoordig minder hoffelijk zijn. Wat het respecteren van stoptekens betreft zien we eenzelfde trend (cfr. Figure 40, p. 143).

Conclusie: deze trends suggereren dat de niet te ontkennen afname van de ‘thin trust’ die in onderzoeken naar boven komt, ons gedrag tgo vreemden heeft beïnvloed.

Een mogelijke maatstaf voor eerlijkheid en betrouwbaarheid is de criminaliteitsratio. In de VS zien we een sterke toename midden de ’60, parallel aan de daling van andere indicatoren van sociaal kapitaal, vertrouwen, betrouwbaarheid. Aan de ene kant kan die criminaliteit een gevolg zijn van de verzwakte sociale controle; Anderzijds is criminaliteit ook nauw verbonden met andere factoren, zoals het aandeel van jeugd in de nationale bevolking, de evolutie van druggebruik, … Uiteindelijk lijkt slechts een fractie van de stijging in criminaliteit sinds ’60 te wijten aan een algemene daling in nationale eerlijkheid.

Alternatieven voor sociale netwerken:Zoals vroeger vermeld, zijn ‘rechtsregels’ (formele contracten, rechtbanken, …) één alternatief voor ‘generalized reciprocity’ en maatschappelijk ingebedde eerlijkheid. We zouden dan verwachten dat de daling in ‘thin trust’ wordt gecompenseerd door een groter vertrouwen in de wet als basis voor samenwerking. Deze hypothese kan getest worden aan de hand van een analyse van de nationale investeringen in het rechtssysteem.

We stellen vast dat het aandeel transactiekosten verbonden aan dit rechtssysteem, doorheen de eeuw enorm steeg in de Amerikaanse economie (cfr. Figure 42, p. 145). Vooral het relatieve aantal advocaten (per 1000 werkenden) steeg sinds 1970: meer dan een verdubbeling! Economische, sociale en culturele revoluties lijken hier geen rol te spelen, gezien de relatieve stabiliteit van het aantal advocaten tussen 1900 en 1970. Wat ordehandhaving betreft zien we een gelijkaardige trend, hoewel minder ingrijpend (+40% politie-agenten, … in 1995 tgo. 1970). Deze trends blijven bestaan na eliminatie van bevolkingsgroei en economische groei. Ook na vergelijking met de trend in andere beroepen (bvb dokters) blijft deze trend overeind.

De verklaring voor deze explosieve toename van maatschappelijke investering in formele mechanismen van sociale controle en conflictregulering is niet helemaal duidelijk.

- Aan de aanbodzijde is niet zozeer het succes van rechtsopleidingen belangrijk, wel waarom wij plots zoveel meer investeren in deze sector.

- Aan de vraagzijde speelt ongetwijfeld de stijging van de criminaliteitscijfers sinds 1970 een belangrijke rol. Anderzijds is de wetgeving niet zozeer toegenomen, zodat criminaliteit op zich een eerder beperkte rol speelde.

Andere verklaringen?

- Volgens sommigen is de groei te wijten aan de welvaartsstijging en de toegenomen socio-economische complexiteit, hoewel het moeilijk te verklaren is waarom de trend dan niet voor 1970 zichtbaar was;

- De toegenomen overheidsregulering kan een gedeeltelijke verklaring bieden, hoewel het opvalt dat het corporatisme ten tijde van de New Deal en het ontstaan van onze welvaartsstaat geen vergelijkbaar effect veroorzaakten tijdens de ’30-’40;

- Het toegenomen aantal echtscheidingen tijdens de ’70 verklaart ook ten dele;- ‘Preventive lawyering’ is verantwoordelijk voor het grootste deel van de stijging aan de

vraagzijde. Sinds 1970 bleken informele overeenkomsten niet meer te voldoen. Meer en meer moest men overeenkomsten ‘op papier’ kunnen zetten. ‘Urban sprawl’: groei voorsteden laat band met lokale gemeenschap vervagen.

Conclusie:Het toegenomen bedrag dat we spenderen aan advocaten om geschillen te anticiperen en op te lossen, is misschien wel een van de meest opvallende indicatoren van het afbrokkelen van ons sociaal web. We vertrouwen meer en meer op formele instituties en bovenal op de wet, om te bekomen wat we vroeger bekwamen via informele netwerken, versterkt door ‘generalized reciprocity’ – dit is, door sociaal kapitaal.

Chapter 15: What killed civic engagement? Summing up. (276-284)Wat heeft ertoe geleid dat er een terugval is in het zich engageren voor de samenleving?

(gebaseerd op Amerikaans onderzoek)

1) De traditionele Amerikaanse familiestructuur bestaat niet meer.

15

Page 16: Facultair Overlegorgaan Sociale Wetenschappen … · Web viewSolidariteit op de werkvloer maakt deel uit van een ruimere sociale solidariteit 2) Kerk en religiositeit: Georganiseerde

DAAN DELESPAUL 2017-2018

De familie wordt gezien als een belangrijke leverancier van sociaal kapitaal. Het is blijkbaar zo dat met de verandering van de familiestructuur er een verminderde participatie is van de burger in het verenigingsleven, dus dat er minder sociaal kapitaal meegegeven wordt.

Op zoek naar een verklaring hiervoor wordt de vraag gesteld of die niet traditionele structuren leiden tot minder maatschappelijk engagement. Het antwoord hierop is hoogstwaarschijnlijk; NIET VEEL.

Huwelijk en kinderen veranderen inderdaad het sociale netwerk maar ouders zijn meer actief in schoolverenigingen en verenigingen die het belang van het kinderen aanhangen. In andere soorten verenigingen zijn ouders en gehuwden minder geïnteresseerd. Ze plannen meer activiteiten bij hen thuis maar hebben minder informele contacten met hun buren en vrienden.

Het huwelijkscijfer is wel gedaald en de sociale activiteit is verschoven maar heeft geen effect op de mate van participatie. Het hoger scheidingscijfer zorgt ook al niet voor minder participatie.

Er is wel een link tussen de verandering van de familiestructuur en het verminderen van de kerkgang maar het is niet duidelijk welke van de twee elkaar beïnvloed heeft. Hier stelt het probleem van causaliteit zich.

2) Speelt ras een rol in de afbrokkeling van sociaal kapitaal?

Men ziet namelijk de een grote vermindering van sociale verbondenheid wanneer het grootste succes van de burgerlijke rechten in de ’60 plaatsvindt.

NEEN, zwarten behoorden tot de jaren ’80 het meest tot verenigingen maar dan wel tot etnische en religieuze en niet tot andere. De stelling dat de afbrokkeling van sociaal kapitaal alle rassen heeft beïnvloed gaat ook niet op want de daling van lid zijn van een vereniging van zwarten en blanken (al dan niet voor segregatie) blijft gelijk. (heel vaag) maar ras heeft geen invloed

3) Draagt de groei van de welvaartstaat bij tot de vermindering van sociaal kapitaal?

Hoe dit onderzoeken? Door een onderzoek te voeren naar sociaal engagement en beleid in verschillende jurisdicties en na te gaan of veel overheidsoptreden leidt tot een daling van sociaal kapitaal. Het antwoord daarop is NEEN. Cross-nationaal heeft onderzoek wel uitgewezen dat sociaal kapitaal het meest aanwezig is in de beste welvaartsstaten. Maar hier stelt zich weer een probleem van de causaliteit. Wat veroorzaakt wat?

(uitleg bij grafiek )

Er zijn slechts twee zaken veranderd: uitgaven voor defensie zijn gedaald en uitgaven voor het statelijk en plaatselijk niveau gestegen. En er zijn slechts twee zaken niet veranderd: op federaal en lokaal vlak wordt er evenveel budget besteed en de grootte van het budget aan binnenlands beleid op federaal niveau is niet veranderd.

Sociaal kapitaal steeg echter wel tussen ’47-’65 maar daalde dan waar tussen ’65-’98.

Het is dus fout te stellen dat de burgerlijke participatie daalt als er een hoger uitgavepatroon is op federaal niveau.

Samenvattend

Verminderd maatschappelijk engagement is te wijten aan:

- geld en tijd ( 10% )- suburbanisatie, onregelmatige uitbreiding ( 10% )- TV ( 25% )- Generationele verandering is de grootste verklarende factor maar niet voor alle niveaus

van vermindering tot engagement. Dit verklaart vb wel de verminderde kerkgang goed maar niet echt het minder bezoeken van vrienden.

- Overlapping tussen generationele verandering en lange termijneffecten van TV-kijken ( 15% )

- Nog factoren waarvan we echter niet weten dat ze zorgen voor verminderd engagement in de samenleving

De specifieke effecten variëren wel binnen de verschillende vormen van gemeenschapsbetrokkenheid.

16

Page 17: Facultair Overlegorgaan Sociale Wetenschappen … · Web viewSolidariteit op de werkvloer maakt deel uit van een ruimere sociale solidariteit 2) Kerk en religiositeit: Georganiseerde

DAAN DELESPAUL 2017-2018

6. Robert Inglehart (2008): Changing Values among Western Publics from 1970-2006 (p. 130-

146)

Zie nota’s tekst en notities

VertrekpuntSurvey 1971: duidelijk verschil waardepatroon (materieel versus postmaterieel) naar leeftijd:

Materieel = nadruk op economische zekerheid en groei, bestrijden van prijsstijgingen, handhaven van orde en misdaadbestrijding

Postmaterieel = vrijheid van meningsuiting, mensen moeten meer inspraak krijgen in overheid en werk, een maatschappij waar levenskwaliteit belangrijk is.

Voorspelling Inglehart: geen levenscyclus (meer materialistisch wanneer je ouder wordt, maar stabiele generatieverschillen met belangrijke gevolgen voor toekomst (verschil tussen geboortecohorten). Het verschil is m.a.w. intergenerationeel en niet zomaar verschillend per leeftijd.

Twee kernhypothesen:

- Scarcity hypothesis: (schaarste) waarden die mensen aanhangen zijn verbonden met de mate van welvaart in hun samenleving. “highest value on the most pressing needs”. (vooral tijdens kinderjaren)

- Persistence/socialisation hypothesis: stabiliteit van waardenpatronen na jeugdsocialisatie

Economie en waardenStijgende bestaanszekerheid leidt tot andere waarden:

- sinds WOII: ongekende groei (geen zorgen over geen eten, of werk te verliezen).

- maar dit gaat over een subjectief gevoel (≠pro capita)

(je hebt een aantal existentiële zekerheden ↔ VS: ziekteverzekering hangt samen met het wel of niet hebben van een baan)

Bv existentiële zekerheden over spaarcenten bij Fortis of Dexia omdat Leterme de spaarcenten heeft gewaarborgd indien je bank failliet zou gaan.

“There is no one-to-one relationship between socio-economic development and the prevalence of post-materialist values, for these values reflect one’s subjective sense of security, not simply one’s objective economic level.”

Hoe meten?Surveyonderzoek: mensen vragen naar waardeprioriteiten => figuur p.134 Vervolgens indelen volgens een typologie. Zo een eenmalige meting is echter niet voldoende: kan ook levenscyclus effect zijn. Maar het is geen levenscyclus: mensen veranderen niet, of toch niet zo fel, van waarden naarmate zij ouder worden. Hoe dan wel aantonen van generatieverschillen? cohorte analyse!

Cohorte Analyse (time-series analysis):

- data die over zeer lange periode zijn verworven

- regelmatige tijdstippen van opname

- grote aantallen respondenten wegens deelsamples

17

Page 18: Facultair Overlegorgaan Sociale Wetenschappen … · Web viewSolidariteit op de werkvloer maakt deel uit van een ruimere sociale solidariteit 2) Kerk en religiositeit: Georganiseerde

DAAN DELESPAUL 2017-2018

Bredere inbedding in waardeveranderingPostmateriële shift zit ingebed in verschuiving in vele culturele waarden (in ‘advanced industrial societies’) M.a.w., er zijn heel veel culturele waarden veranderd op een hele korte tijd: religiositeit, gender rollen, seksuele voorkeur.

hangen deze waarden samen? = factoranalyse (factor analyse is een methode om uit een reeks factoren een patroon te vinden)

Een factoranalyse op een hele reeks waarde-items toont aan dat er twee waardedimensies zijn die elkaar tegenstellen:

*Traditioneel versus seculier-rationeel

*Overlevings- versus zelfexpressie waarden

Dimensie materieel vs. postmaterieel hangt met de laatste samen: rijke landen zijn meer geneigd zelfexpressie te omarmen door bv. democratisch te zijn en hun kind minder streng op te voeden. culturele waarden hangen (sterk) samen met de zelfexpressie waarden.

Indien men weet of je mat of post-mat bent, kan men de rest voorspellen.

ZelfexpressieHangt onder meer samen met:

- Visie op rol van vrouwen

- tolerantie naar out-groups

- waarden die men doorgeeft

- politieke activiteit (zie andere tekst Inglehart)

- minder respect voor autoriteit (onder onzekerheid doe je dit wel) =>bv. werkzekerheid

Dit betekent dat er ook een link is met democratie en afwijzen van autoritarisme

Generationele shiftOok hier zien we generationele shift.

- betekent niet dat aandacht voor materiële verdwijnt

- wel treden nieuwe waarden die zelfexpressie uiten op de voorgrond: ecologie, abortus, homohuwelijk.

- tegelijk afname social class voting, partijlidmaatschap, etc...

Conclusie: - mensen zijn ‘fed up’ (beu) met politiek

- mensen willen te veel en zijn dus niet tevreden (zie ook Huntington & Crozier)

- politici zijn corrupter

Inglehart: mogelijk... maar is vooral een uiting van dalende eerbied voor autoriteiten in ‘advanced industrial societies’

Modernisatie Inglehart stelt een niet-lineaire visie op modernisatie voor:

- eerste stap is van pre-modern naar modern door industrialisatie

- tweede stap is van modern naar postmodern.

Eerste fase leidt tot secularisatie en een rationeler wereldbeeld en bureaucratisering, niet tot een daling voor authoritarisme (integendeel...). Er wordt zelfs een grotere rol voor de staat weggelegd die de rol van God in de seculiere wereld moet overnemen.

Marx: eerste fase duurt eindeloos. Als deze periode lang zou duren worden arbeiders slaven.

Inglehart: er is een knik die Marx niet ‘ziet’: arbeiders gaan goed verdienen en krijgen andere waarden (=stap 2)

1e stap: 1950: Phillipssite in Leuven= tv’s maken; 2008: India, China= Ipods maken.

Mensen kwamen toen ook al op straat, maar voor een ANDERE staat, een sterke staat die de bevolking zou leiden naar een betere wereld.

18

Page 19: Facultair Overlegorgaan Sociale Wetenschappen … · Web viewSolidariteit op de werkvloer maakt deel uit van een ruimere sociale solidariteit 2) Kerk en religiositeit: Georganiseerde

DAAN DELESPAUL 2017-2018

7. Robert Inglehart (1999): Postmodernization erodes Respect for Authority but increases

Support for Democracy (p. 236-265)De voedingsbodem van het postmodernisme ligt in de geïndustrialiseerde maatschappij. Mensen krijgen erdoor ook een kritischere kijk op het de politieke leiders. Het respect voor gezag vermindert zienderogen maar de ondersteuning voor de democratie wordt alleen maar groter. Het verlies aan respect voor de gezagshebbende instanties zorgt er wel voor dat beleid bepalen en voeren er heel moeilijk op geworden is. Er is dus geen verlies aan vertrouwen in democratische waarden, want mensen zijn nu ook meer bereid om aan politieke participatie te doen. Deze participatie maakt de democratie meer democratisch maar de elite die de macht in handen heeft kan daardoor wel moeilijker een beleid gaan bepalen dat een positieve evaluatie zal krijgen.

Modernisme, postmodernisme en culturele verandering Waarom verliest men het vertrouwen in het gezag?

Hypothese: door economische ontwikkeling ontstaan er functionele veranderingen in de waarden en het geloofssysteem dat de massa aanhangt.

De samenleving industrialiseert dus waardoor er veranderingen komen op het vlak van sociale mobiliteit, gender verschillen dalen…

De evolutie in het modernisme verloopt niet lineair; er is een trend van modernisme, die wereldwijde materiële rationaliteit tot stand bracht, naar postmodernisme waarin de nadruk komt te liggen op individuele zelfontplooiing.

Weber: het wereldbeeld georiënteerd op religie is geëvolueerd naar een rationeel, legaal wereldbeeld

1) SeculariseringDe samenleving gebaseerd op agricultuur had een religie nodig om een mate van zekerheid te creëren in een onzekere omgeving. Wij daarentegen hebben in onze samenleving een groot zekerheidsgevoel.

2) Bureaucratisering Er is een toename van rationele organisaties die gericht zijn om het zo efficiënt mogelijk aanwenden van de middelen en zo doelgericht mogelijk te werken. De plaats van God wordt ingenomen door de staat. Ze krijgt een groeiende politieke, economische en sociale rol.

Postmodernistische verschuiving De bureaucratie bereikt zijn limiet waardoor de verandering die aan gang is in een andere nieuwe richting opschuift.

1) modernisme: traditioneel gezag gebaseerd op religie en gemeenschappelijke waarden (familiale en religieuze instellingen)

Rationeel, legaal gezag gebaseerd op individuele ontwikkeling, motivatie, economische ontwikkeling (politieke instellingen)

2) postmodernisme: door de grote economische zekerheid komt de nadruk te liggen op het individu (vrienden, ontspanning, …)

Het respect voor gezag daalt, t.v.v. een grotere mogelijkheid om tot autonome zelfontplooiing te komen.

Optimaliseren van het subjectieve welzijn = postmaterialisme (cross-cultureel)

Kenmerken:

- tolerantie, verbeelding, - geen nadruk op werken- (weinig geloof dat wetenschap voordelen heeft voor het menszijn)- hechten meer belang aan ecologie en daarbij minder aan technologie

Het modernisme en het postmodernisme zijn wel allebei gerelateerd aan economische groei want als er economische ontwikkeling is dan krijgen de mensen een ander zicht op hoe ze de wereld georganiseerd willen zien.

19

Page 20: Facultair Overlegorgaan Sociale Wetenschappen … · Web viewSolidariteit op de werkvloer maakt deel uit van een ruimere sociale solidariteit 2) Kerk en religiositeit: Georganiseerde

DAAN DELESPAUL 2017-2018

Voorbeeld: gezag Wanneer economische ontwikkeling plaatsvindt, vermindert het respect voor gezag. De grafiek toont dit aan: landen met een laag BNP, die dus nog niet geïndustrialiseerd zijn en geen grote economische ontwikkeling ondergaan hebben vinden meer respect voor gezag een goeie zaak.

1 opmerking bij de grafiek: men kan niet enkel afgaan op de economie want Taiwan, Zuid-Korea en China vinden meer respect voor gezag geen goeie zaak terwijl hun BNP wel laag ligt.

Gezagsreflex en verschuiving naar postmodernismeBestaat er een algemene aliënatie?

Hypothese: lage opkomst bij verkiezingen zorgt voor vervreemding van het systeem

NIET WAAR: Westers samenlevingen zijn niet politiek apathisch want tegenwoordig is er een grote tendens tot meer politieke participatie.

Niet alle instituties verliezen aan vertrouwen maar enkel de gezagshebbende instituties.

2 visies: materialisme: veel vertrouwen in gezagshebbende instituties

postmaterialisme: weinig vertrouwen in gezagshebbende instituties

Als de maatschappij snel veranderingen ondergaat, ontstaat er een groter onzekerheidsgevoel waardoor er een reflex ontstaat naar autoriteit (die zekerheid kan bieden). Daarbij moet gezegd worden dat eerbied voor gezag een hoge persoonlijke kost heeft en er dus minder mogelijkheid bestaat tot zelfontplooiingen. Men is dat blijkbaar wel bereid op te geven als men zich in een bedreigde situatie voelt terechtkomen.

Er is een verschuiving aan de gang die weinig vertrouwen stelt in de kerk en de bureaucratie.

Verminderd vertrouwen in hiërarchische instellingenTwee factoren maakten de industriële revolutie tot een succes:

- productie aan de lopende band- bureaucratisch model

Dit zijn hiërarchisch, centraal georganiseerde instellingen waar geen plaats is voor persoonlijk vrijheid. Postmodernisten hebben weinig vertrouwen in zo’n instellingen en ze verschuiven ook van respect voor staatsgezag of respect voor traditioneel gezag naar respect voor gezag in het algemeen.

De idee minder vertrouwen te kunnen hebben in politieke leiders zit verweven in de geïndustrialiseerde samenlevingen. Dit hangt samen met de specifieke waarden die postmaterialisten hoog achten. Het minder vertrouwen hebben in gezag over de tijd heen noemt men het proces van intergenerationele verandering.

(nu volgt analyse van 3 grafieken)

1) In geïndustrialiseerde samenlevingen hebben jongeren minder respect voor het gezag dan ouderen. In minder ontwikkelde landen is dit niet het geval; jongeren en ouderen hebben een quasi gelijk niveau van respect voor het gezag.

de tendens van ouderen om meer respect te hebben voor het gezag hangt niet af van een bepaald stadium in hun levenscyclus maar van specifieke historische veranderingen in specifieke samenlevingen.

2) Laaggeschoolden hebben meer respect voor het gezag dan hooggeschoolden in geïndustrialiseerde landen. Zo’n effect bestaat niet in minder ontwikkelde landen. Daar neigen de hooggeschoolden eerder meer respect te hebben voor gezag omdat zij waarschijnlijk de elite zijn die de macht in handen heeft en deze ook niet wil afstaan.

3) Materialistische en postmaterialistische waarden correleren meer met het hebben van een attitude pro gezag dan leeftijd of opleiding.

Postmaterialisten die te vinden zijn in geïndustrialiseerde samenlevingen hebben weinig vertrouwen in gezaghebbende en hiërarchische instanties zoals kerk, politie, leger.

De intergenerationele verandering vindt nu plaatst doordat de jonge cohortes de oude vervangen. Meer postmaterialistisch vervangen meer materialistische cohortes dus het vertrouwen in hiërarchische instanties zal over de tijd blijven dalen.

20

Page 21: Facultair Overlegorgaan Sociale Wetenschappen … · Web viewSolidariteit op de werkvloer maakt deel uit van een ruimere sociale solidariteit 2) Kerk en religiositeit: Georganiseerde

DAAN DELESPAUL 2017-2018

Normen over gezag: voorspellingen en veranderingenEr bestaat een graduele verschuiving naar postmaterialistische waarden

gebaseerd op het populatie vervangingsmodel: cfr intergenerationele verandering

voorspelling: minder respect voor het gezag: 4 à 5 procentpunten over 9 jaar

Voor 78% van de onderzochte landen gaat dit op. Sommige landen hebben een te hoge verschuiving maar dit heeft te maken met situatie gebonden factoren bij dat land, factoren die dus buiten het model liggen.

Postmaterialisten hebben een kritischere kijk op wat politici doen en zullen ze er ook op afrekenen als genomen beslissingen hen niet bevallen. Hoewel zij meer kans hebben om hun voorkeuren door te laten wegen zullen zij toch kritischer zijn tegenover de politici.

Paradox: Hoewel de bevolking in geïndustrialiseerde landen beter af is dan de bevolking in ontwikkelingslanden putten ze geen grote tevredenheid uit hun politieke systemen. Ze staan gewoon veel kritischer tegenover het beleid dat gevoerd wordt en trachten daar actief aan deel te nemen.

Afbrokkeling van vertrouwen in hiërarchische instellingenVoorspelling: alle oriëntaties gelinkt aan postmaterialistische waarden zullen meer verspreid worden

verandering in politieke waarden en maatschappelijke waarden

Er is een afbrokkeling voor institutionele autoriteit en een stijging bij de bevolking om politiek te participeren. De elites hebben het moeilijk te verduren omdat de bevolking door zelf te participeren de touwtjes in handen neemt waardoor minder speelruimte overblijft voor de beslissende elite.

negatief: instituties zouden zodanig uitgehold worden dat ze niet meer in staat zijn te reageren op een nationale ramp

positief: - op lange termijn is er een daling van nationalistische tendenzen

- grotere democratisering

Wegebben van vertrouwen in hiërarchische instellingen Is er een verschuiving te zien in de tijd naar de manier waarop jongere en meer postmaterialistische mensen naar de wereld kijken?

JA: in ’97 is er minder vertrouwen in de gezagsinstanties dan in ‘81

Verklaring: postmaterialistische waarden worden sterk gelinkt aan weinig vertrouwen hebben in politiek, kerk en leger

Voorspelling: het vertrouwen in deze instellingen zal nog dalen

1 grote uitzondering: ’81-’90: vergroten van het vertrouwen in corporaties

Verklaring hiervoor is het ineenstorten van socialistische staatsgeleide economieën. Privé ondernemingschap werd dan automatisch als positief ervaren.

Ontwikkelde landen staan negatief tegenover een staatsgeleide economie omdat de staat hier niet meer gezien wordt als de ultieme oplossing voor alle problemen.

Ondermijnen deze trends de ondersteuning voor democratische instellingen?

NEEN, ze ondermijnen de ondersteuning voor hiërarchisch gezag wat beleid voeren wel moeilijker maakt. De bevolking van de meest postmaterialistische samenlevingen keuren het sterkst autoritaire regeringen af.

postmodernisme leidt tot een afname aan respect voor het gezag maar tot een grotere steun voor democratie

21

Page 22: Facultair Overlegorgaan Sociale Wetenschappen … · Web viewSolidariteit op de werkvloer maakt deel uit van een ruimere sociale solidariteit 2) Kerk en religiositeit: Georganiseerde

DAAN DELESPAUL 2017-2018

8. Pierre Rosanvallon (2006) : La Contre-Démocratie.

La Politique à l'Âge de la Défiance (p.1-32, 63-79, 295-322) ENKEL DEEL 1!

Het blijft maar de vraag of vertrouwen essentieel is binnen een democratie. Rosanvallon meent dat een functionerende democratie juist nood heeft aan wantrouwige burgers die het systeem

controleren in een vorm van tegendemocratie.

Defiance et democratie (introduction)Het probleem van de politiek in onze tijd is de erosie van het vertrouwen van burgers in hun beleidsmakers en hun politieke instellingen. Nagelnieuwe democratische regimes ontsnappen hier niet aan (zie Oost-Europa, Azie, Latijns-Amerika). Veel studies rond gedaan en vaak komen zaken als stijgend individualisme, het herclaimen van de privé-sfeer en een daling in politieke wil naar boven. Algemene 'déclin du politique'.

Historisch gezien uit de democratie zich altijd tweezijdig: als een belofte en als een probleem. Een belofte op de realisatie van zowel gelijkheid als autonomie, en een probleem in het waarmaken van dat idealisme. Resultaat is dat de democratie ook steeds heeft moeten leven met mensen en mogendheden die haar in vraag stellen, en instellingen die dat mogelijk maken.

La societe de defianceDe theorie van democratische representatieve overheden is gebaseerd op het electorale mechanisme: legitimiteit en vertrouwen. Vertrouwen is hier een veel vager begrip van legitimiteit en heeft eigenlijk drie functies: 1. de kwaliteit van legitimiteit vergroten door er een morele (integriteit) en 2. een substantiële (de zorg om het gemeenschappelijk welzijn) dimensie aan toe te voegen. 3. Ze biedt een zekere garantie dat de legitimiteit ook in de toekomst zal voortbestaan, omdat het vertrouwen van de bevolking vereist is om een democratie te doen werken.

Om de werking van een democratie te begrijpen moet men in gedachten houden dat er steeds twee verschillende bewegingen in het spel zijn: de werking en de problemen van de gevestigde democratische instellingen enerzijds, en het bestaan van wantrouwen en eventuele weerstand anderzijds. De eerste dimensie is het domein van geschiedkundigen en politicologen, het tweede dat van sociologen. Rosanvallon stelt dat het hier gaat over wantrouwen dat globaal en op een coherente manier wordt uitgedrukt en dus een relevante uitbreiding vormt op het politieke systeem. (cfr: cursus politiek en sociaal denken; “the price of freedom is eternal vigilance”) Dit om een terugkeer van het despotisme te voorkomen (met ancien regime en Franse revolutie in gedachten). Constant spreekt hier zelfs van een 'surveillance van de haat'. Hij stelt dat het het vertrouwen in de democratie zelf is dat men moet in het oog houden en beperken, want een blind vertrouwen maakt soms dat eerlijke mensen meewerken aan bloedige ondernemingen. Een 'defiance liberal' kan op die manier een macht tot preventie worden.

Er is echter nog een andere invulling van dat wantrouwen dat voor het boek belangrijker is, met name het wantrouwen ontstaan in het post-totalitaire tijdperk dat zich op een veel veelzijdigere manier manifesteert. Dit is een 'tegen-democratie' die bestaat uit drie modaliteiten: les pouvoirs de surveillance, les formes d'empechement en les mises a l'epreuve d'un jugement. Deze contra-democratie is niet noodzakelijk tegengesteld aan de conventionele democratie, ze staat er gewoon tegenover als georganiseerde, indirecte macht vanuit het volk, bij wijze van tegenstroom om een eigen sturing en invloed uit te oefenen op het beleid.

Deze 'samenleving van wantrouwen' wordt door drie factoren tot leven gebracht:

1. Wetenschap: technologische en wetenschappelijke vooruitgang heeft serieuze ballast en risico's met zich meegebracht die maken dat mensen met twijfel en wantrouwen naar de toekomst kijken, terwijl ze zelf als burger nauwelijks directe grenzen kunnen stellen aan de ontwikkelingen. Het enige dat de burger hier kan doen is meer waakzaamheid en controle vragen.

2. Economie: doordat het spel van enorm veel onvoorspelbare factoren en interacties in rekening brengt, is het voor een staat moeilijk geworden een effectief beleid te voeren op economisch vlak. De burger heeft hier kritiek op, maar wordt met hetzelfde probleem geconfronteerd.

3. Sociologie: het dalende interpersoonlijke vertrouwen correleert exact met het

22

Page 23: Facultair Overlegorgaan Sociale Wetenschappen … · Web viewSolidariteit op de werkvloer maakt deel uit van een ruimere sociale solidariteit 2) Kerk en religiositeit: Georganiseerde

DAAN DELESPAUL 2017-2018

dalende vertrouwen in politieke instellingen. Hierdoor kunnen we het structurele wantrouwen en het democratische wantrouwen veralgemenen tot een soort 'samenleving van algemeen wantrouwen'. Het is tegen deze achtergrond dat men de veranderingen in de democratie moet bekijken.

Les trois dimensions de la contre-democratieHistorisch gezien bestaat de volkssoevereiniteit uit twee delen: stemrecht en permanente controle. De omstandigheden van een parlementaire democratie zijn echter zo veranderd en complexer geworden dat er nauwelijks nog sprake is van een directe controle van de burger op zijn/haar vertegenwoordiger tussen de verkiezingen door. Hierop vormt de tegen-democratie een correctie, waardoor op die manier de levensduur van een democratie al eens aanzienlijk verlengd kan worden. De waakzaamheid van het volk werd in 1789 gevierd als de grote remedie tegen institutionele wantoestanden, zelfs als een remedie tegen het imperatief mandaat.

Vandaag komen daar nog twee elementen bij en bestaat de contra-democratie uit drie modaliteiten: la vigilance (waakzaamheid) la denonciation (aanklacht) en la notation (waardering). Alledrie dragen ze bij aan het in vraag stellen van de sociale legitimiteit van een regime, en dit op initiatief van zowel individuen als organisaties.

Er is altijd de twijfel rond de effectiviteit van een actie en duurzamere lange-termijnsmethodes hieronder lijden, maar anderzijds is er wel het positieve dat een negatieve coalitie veel gemakkelijker te organiseren is dan een positieve, en zeer heterogene groepen elkaar hierin kunnen vinden. Dit maakt ze wel kwetsbaarder.

Verder is ook de volksjury een vorm van contra-democratie, zij het dan wel een tegen-democratie die haar plaats heeft gevonden in het systeem. Zo zijn we van een democratie van confrontatie geëvolueerd naar een democratie van imputatie.

Op die manier zijn er aan het sociaal-contract (zie sociale contract-theorie) drie figuren toegevoegd: de peuple-surveillant, de peuple-veto en de peuple-juge. Het is ook de bedoeling de definitie van burgerschap in deze richting steeds verder uit te breiden. Rousseau en Hirschman vermelden dit reeds in hun essays.

Le mythe du citoyen passifDe afgelopen twintig jaar heeft men in quasi alle landen een stijging in politieke onthouding vastgesteld. Dit dient men echter met voorzichtigheid te interpreteren en het te situeren in een breder denkkader waarin het concept van burgerschap grote veranderingen onderging. De politieke wetenschappen erkent al lang de toename en het belang van 'niet-conventionele participatiemethodes', die suggereren dat we helemaal niet in een periode van politieke apathie beland zijn. Het past dan ook beter te spreken van een mutatie dan van een daling van het burgerschap. De grote representatieve instelling zien hun rol bedreigd door een veelheid aan kleine ad hoc belangenorganisaties, en burgers hebben veel meer kanalen waarlangs ze hun grieven kwijt kunnen. Het fenomeen van politieke onthouding en de daling in het vertrouwen moet dan ook in dit licht bestudeerd worden. Er zijn nu drie dimensies van interactie tussen het volk en de politiek: democratie d'expression (meningsuiting), democratie d'implication (betrokkenheid) en democratie d'intervention (interventie) Men neemt geen genoegen meer met louter een verkiezingsstem als uitdrukking van individuele voorkeur en zoekt zijn heil in collectieve bewegingen. De mythe van de passieve burger slaat dan ook op het feit dat de erosie van de verkiezingsdemocratie ruimschoots gecompenseerd wordt door deze nieuwe vormen van participatie, en de burger vandaag dus allesbehalve passief is. Het geheel wordt dan een mengvorm van protest-politiek en civiele politiek, en zou zijn plaats moeten hebben in het concept van democratie.

Depolitisation ou impolitique?Een andere kwestie in verband met de contra-democratie blijkt van belang: niet de politieke passiviteit, maar het soms apolitieke karakter van de bewegingen kan voor problemen te zorgen. Men richt zich namelijk niet meer alleen op de politieke instellingen, maar op de maatschappij en de geglobaliseerde wereld als geheel. De activiteit blijft democratisch, maar het effect is niet meer alleen politiek. Op die manier ontsnapt ze ook aan alle traditionele politieke denkkaders. Verder is het ook zo dat de contra-democratie te kampen heeft met een probleem van duidelijkheid en coherentie. De beweging is namelijk enorm heterogeen en gefragmenteerd.

Volgens Rosenvallon is dit exact de aard die de hedendaagse democratieën vanaf nu zal kenmerken.

23

Page 24: Facultair Overlegorgaan Sociale Wetenschappen … · Web viewSolidariteit op de werkvloer maakt deel uit van een ruimere sociale solidariteit 2) Kerk en religiositeit: Georganiseerde

DAAN DELESPAUL 2017-2018

Relire l'histoire de la democratieRosenvallon wijst er ook op dat de contra-democratie geen nieuw fenomeen is, aangezien er ten tijde van het absolutisme al bewegingen bestonden die de bestaande situatie in vraag stelden en hiermee de eerste stap zetten in het proces van de menselijke emancipatie (ook al had men destijds niet het besef dat een volkssoevereiniteit ooit realiteit zou worden). Dit brengt ook het feit onder de aandacht dat de democratie zelf een ideaaltype is en dat de verschillende strekkingen binnen de maatschappij (liberalisme, democratie, de oude en en het nieuwe) niet zijn opgehouden met de degens te kruisen. De contra-democratie is in zijn essentie vraag, sanctie en contestatie en aangezien zij net zo historisch en natuurlijk is als de institutionele structuren, mogen die twee niet gescheiden worden. Men moet politiek hier ook zien als een constant bewegend gegeven, wat de appreciatie voor permanente waakzaamheid nog groter maakt. Men moet dan ook ten alle tijde vermijden dat bepaalde waarden voor universeel worden aanzien, of bepaalde mechanismen als heilig. Ook veralgemeningen zijn uit den boze.

NIET COMPLEET!!

9. R. Dalton & C. Welzel (2014) : The Civic Culture Transformed. From Allegeant to Assertive

Citizens. (p.1-18, 282-306) Belangrijke evoluties in de 20ste eeuw tonen aan dat het traditionele allegeant model van politieke cultuur niet het enige is dat een gezonde democratie kan faciliteren. Door de overgang naar post-

materiële waarden laten burgers zich nu eerder kenmerken door een assertive model.

Political Culture and Value ChangeVijftig jaar geleden deden Almond en Verba studie naar “civicness”. Destijds was er geen goed medium om hun beweringen te funderen, tegenwoordig bezitten we echter heel wat cijfermateriaal. Almond en Verba meenden dat voor een democratie een “allegeant model” nodig was zoals in de VSA en VK. Inglehart heeft aangetoond dat de publieke waarden dramatisch zijn veranderd ten opzichte van Almond en Verba naar meer assertive en zelf-expressieve waarden. We veranderen dus van loyauteit naar kritische burgers. Ook waren waarden van Almond en Verba erg Westers gericht: in ontwikkelingslanden liggen ze helemaal anders.

Evolution of political culture research

Centrale assumptie van Almond en Verba was dat de politieke cultuur van een land overeen moest komen met de structuren van het politiek systeem van dat land om tot een stabiele democratie te komen. De ideale burger is loyaal en participeert. Door onderwijs en sociale modernisering konden de juiste waarden worden aangeleerd, anderzijds leidt dit ook tot meer autonomie en een kritischere houding ten aanzien van het bestuur.

Kritiek

Demonstranten en opstandelingen in de jaren ’60 tonen dat loyauteit niet essentieel is om in democratie te geloven. Onderzoek wees uit dat dit type zelfs eerder geneigd was de democratie te steunen. Als reactie hierop schreven Almond en Verba Civic Culture Revisited in 1980: hun conclusie was dat de civic culture een flexibele en meerlagige variabele is die reageert op structurele verandering. Inglehart verbond die anti-establishment gevoelens aan postmaterialistische waarden. Zo daalde het participatieniveau en het vertrouwen in de democratie, zonder dat de steun aan de democratie zelf afnam. Dit toont dat een democratie perfect kan overleven met andere waarden, het allegeant model kan dus ook vervangen worden door een assertive model.

From Allegiant to Assertive Citizens

Genuanceerd beeld van een land haar politieke cultuur en het regime. Het blijkt dat overal ter wereld steun voor democratie is of een land nu democratisch is of niet.

An overview of our findings

Inglehart toonde al dat burgers meer waarden zijn beginnen hechten aan postmaterialistische waarden. Nevitte voegde daaraan toe dat waarden met betrekking tot gehoorzaamheid aan autoriteit verschuiven naar meer autonomie in familie, werk en sociale relaties. Toch uit zich dat

24

Page 25: Facultair Overlegorgaan Sociale Wetenschappen … · Web viewSolidariteit op de werkvloer maakt deel uit van een ruimere sociale solidariteit 2) Kerk en religiositeit: Georganiseerde

DAAN DELESPAUL 2017-2018

niet in meer participatie, hoewel het geloof in democratie als staatsbestel juist toeneemt. Burgers blijven dus kritisch ten aanzien van de democratische werking, maar geloven er wel in , deze assertieve houding vindt zijn basis in postmaterialistische waarden.

Deze studie concludeert drie zaken: een overgang naar postmateriële waarden in samenlevingen bij stijging van welvaart, een assertievere houding van burgers en een sterke samenhang tussen post-materiële waarden en steun voor milieu en gendergelijkheid.

Culture and Institution

De politieke cultuur van een land heeft een verregaande invloed op de wijze waarop het land wordt geregeerd. Een democratie installeren zonder democratische cultuur heeft dus geen zin.

The Civic Culture and good governance

Het omgekeerd blijkt evenwel eveneens waar: hoe een land geregeerd wordt heeft net zo goed invloed op de cultuur. Waar allegiant citizens een positieve invloed hebben op het beleid en de legitimiteit is het maar de vraag of assertive citizens dit ook doen.

Measuring Allegiant and Assertive orientations

Allegiant citizens staan bekend om:- vertrouwen in de staatsapparaten en instituten en onder meer politie en politici;- algemeen vertrouwen in anderen en in de democratie en interesse in politiek;- gehoorzaamheid aan de regels.

Assertive citizens hechten meer waarde aan “emancipatieve waarden”:- individuele vrijheden (bv. scheiding en homoseksualiteit);- gelijke kansen;- inspraak van het volk.

Statistisch onderzoek toont geen correlatie tussen allegiant en assertive cultures, daaruit blijkt dat de twee niet tegenovergesteld zijn, maar naast elkaar kunnen voorkomen. Dit neemt niet weg dat het longitudinaal patroon doorgaans is dat wanneer allegiant culture daalt, assertive culture toeneemt. Ontwikkelingslanden kennen vaak een sterke allegiant waarde en een zwakkere assertive, in de ex-Sovietlanden zijn beide culturen laag waar in het westen de assertive culture hoger is dan de allegiant. De Scandinavisch-protestantse landen neigen het meest naar assertive, maar doen het ook m.b.t. allegiant niet slecht.

Predicting governance performance

De vraag is welke invloed de tendens naar assertive culture voor het bestuur heeft. Het blijkt dat een assertive culture meer legitimiteit aan het bestuur geeft, een allegiant culture zorgt dan weer voor een efficiënter bestuur. Het is dus de zaak om beide waarden hoog te houden zoals in de Scandinavische landen. Deze conclusie is echter prematuur nu we merken dat een allegiant cultuur juist vooral gevormd wordt door een efficiëntere overheid. De algemene conclusie is dus, zo blijkt, dat de overgang naar assertive culture helemaal niet problematisch hoeft te zijn. In tegendeel: een assertive culture zorgt voor meer “human empowerment” waardoor ze hun eigen stem meer kunnen laten doorwegen in het politieke debat.

10. M. Hooghe (2015) : Review of ‘The Civic Culture Transformed’ (p.168-170)

Ter herhaling: stabiliteit van een democratie is afhankelijk van een ‘civic culture’. Almond en Verba definieerde die cultuur als een mix tussen een loyale, maar participerende houding. Deze definitie werd uitgedaagd door de bevindingen van Ronald Inglehart die een shift van klassieke naar postmoderne waarden vaststelde, hetgeen ook zo zijn gevolgen had voor de ‘civic culture’ van Almond en Verba.

De auteurs van het boek menen inderdaad dat een ‘civic culture’ essentieel is voor een werkende democratie, maar zijn niet overtuigd van de nood aan een loyale houding van de burgers. De globale steun vanuit de bevolking voor democratie in het algemeen is inderdaad nodig, maar hiervoor moet men niet zozeer loyaal zijn aan de huidige politiek en zijn instellingen. Die steun voor democratie neemt dan ook over de hele wereld toe, terwijl het respect en het vertrouwen in de politiek afneemt en men tot een eerder assertieve houding komt. Een ander basisaspect,

25

Page 26: Facultair Overlegorgaan Sociale Wetenschappen … · Web viewSolidariteit op de werkvloer maakt deel uit van een ruimere sociale solidariteit 2) Kerk en religiositeit: Georganiseerde

DAAN DELESPAUL 2017-2018

vertrouwen, blijkt eveneens overbodig nu juist blijkt dat de bevolking in autoritaire regimes juist meer vertrouwen hebben in de machthebbers.

Dit respect voor de politiek valt niet te rijmen met de toegenomen ‘voice’ die Inglehart vaststelt in de postmoderne waarden. Dit is echter niet problematisch aangezien een kritische ‘civic culture’ net zo goed of zelfs beter kan zorgen voor een stabiele democratie dan een cynische. De vraag blijft desalniettemin toch of deze ‘assertive citizens’ het ideaalbeeld vormen. Ten eerste kan men kritiek opbrengen tegen de grote range die de auteurs gebruiken. Het is inderdaad beter dat men over veel cases heen vergelijkt (in tegenstelling tot Almond en Verba), maar het blijft de vraag of men eenzelfde vraag over de hele wereld kan stellen. Hebben bepaalde begrippen zoals ‘democratie’ immers wel dezelfde betekenis in landen als China en Zweden? Ten tweede maken de auteurs niet helemaal duidelijk of de evolutie naar een assertive culture wel zo goed is. Ze stellen inderdaad een empirische shift vast, maar kunnen moeilijker bewijzen of deze dan wel voordelig is voor de democratie.

11. K. Lehman Schlozman, S. Verba & H. Brady (1999) : Civic Participation and the Equity

Problem (p.427-460) (zie ook notities Politika)

Participatie heeft drie voordelen: het helpt het individu in zijn ontwikkeling, het versterkt bovendien de gemeenschap en democratische waarden en zorgt tenslotte voor gelijke behandeling

van iedereen. Participatie verschilt echter naargelang inkomensgroepen. Rekrutering gebeurt immers voornamelijk voor de meest nuttigen, lageropgeleiden kunnen doorgaans ook niet

financieren.

Why Care about Civic Engagement?

Het eerste voordeel vindt zijn oorsprong in het gedachtengoed van J.S. Mill: participatie maakt de mens onafhankelijker, verantwoordelijker en competenter. Het tweede wordt gesteund door de Tocqueville en zorgt voor sociaal vertrouwen, normen en samenwerking waardoor het makkelijker wordt publieke goederen te produceren. Het derde argument baseert zich op Madison: door de beleidsmakers informatie te geven kunnen van natura verschillende meningen in de samenleving samenkomen. Ook doordat iedereen één stem heeft zal uiteindelijk de balans in het voordeel van iedereen worden gemaakt.

Participatory inequality in America

Er zijn drie redenen om niet te participeren: men kan niet, men wil niet of niemand vroeg erom. In het eerste geval gaat het om een gebrek aan middelen, in het tweede geval om te weinig engagement, hetzij door een gebrek aan interesse, hetzij kennis, hetzij geloof in politiek. In het derde geval faalt het recruteringsproces. Deze drie aspecten: middelen, engagement en recrutering vormen de basis van het Civic Voluntarism Model.

Amerikaanse democratie is in vele opzichten uniek, zeker omwille van de hoge participatiegraad (bv. “campaigning”), gepaard met een lage opkomst. De participatie in de VSA is echter voornamelijk ongelijk verdeeld over de sociale klassen, zeker wat betreft de financiering van campagnes.

Of Time and MoneyTwee variabelen zijn van belang voor participatie: tijd en geld. Tijd verschilt van geld in die zin dat tijd een bovengrens heeft (24 uur per dag), het verschil in inkomen is dus veel groter dan dat in tijdsbesteding. Het is echter niet zo dat iemand met een hoog inkomen meer vrije tijd heeft, over alle klassen heen blijft dat uiteindelijk vrij stabiel. Toch zijn het voornamelijk de rijken die meer participeren. De bovenste 3% is zo verantwoordelijk voor 8% van het aantal uren dat in een campagne gestoken wordt en 35% van de financiële middelen.

Participatory Equality and Government BenefitsWanneer de participatie ongelijk verdeeld is, kan een democratie niet meer de interesses van iedereen behartigen. De overheid hoort ook meer van belangengroepen waarin eerder rijkere burgers zitten dan deze waarin dat minder het geval is.

26

Page 27: Facultair Overlegorgaan Sociale Wetenschappen … · Web viewSolidariteit op de werkvloer maakt deel uit van een ruimere sociale solidariteit 2) Kerk en religiositeit: Georganiseerde

DAAN DELESPAUL 2017-2018

What Messages do They Send?Ook de politieke interesses van de inkomensgroepen variëren sterk. Rijkere groepen focussen meer op postmateriële waarden zoals abortus en de economie, terwijl de lagere klassen zich eerder richten op primaire behoeftes. Omdat de lagere klassen ook minder participeren en communiceren komen deze problemen dan ook minder naar boven bij de politieke elite. Bovendien blijkt ook dat wanneer de leden van een lagere klasse aan politici communiceerden, dit eerder deden vanuit een persoonlijke invalshoed, terwijl de hogere klasse in hun communicatie zou focussen op de maatschappij in zijn geheel.

Overcoming Participatory Inequality through Mobilization

Hoe kunnen we de ongelijkheid in participatie verhelpen? Sociologen hebben zich voor dit onderzoek geïnspireerd op hoe de hogere groepen hun ideeën op de politieke agenda krijgen. Daaruit blijkt dat degenen die anderen betrekken in participatie handelen als “rationale goudzoekers” (rational prospectors) die hun tijd en energie zo efficiënt mogelijk besteden door die mensen aan te spreken waarvan zij verwachten dat zij mee willen participeren en dit goed willen doen. Dat zijn uiteraard degenen die reeds politiek actief zijn en dus niet de achtergestelden.

Who is recruited?De beste predictor voor politieke activiteit blijft opleidingsniveau. Hoogopgeleiden zijn meer geneigd om zowel te participeren als de middelen te hebben om dit te kunnen. Dit maakt ook dat ze meer worden aangesproken voor participatie. De minder hoogopgeleiden zullen nog steeds vaak, hoewel minder dan de hoogopgeleiden, spontaan participeren, maar worden veel minder gerekruteerd. (zie tabel 12-2, p. 447) Als we terugkijken naar het eerste hoofdstuk zagen we dat er drie redenen waren om niet te participeren: men kan niet, men wil niet of men werd niet gevraagd. Doordat blijkt dat de rijksten ook meer middelen hebben om te kunnen participeren kunnen de armsten dus zowel niet aan de eerste als de derde voorwaarde voldoen.

Can Institutions make the Difference?

Vereniging in eender welke sfeer is positief voor de politiek (de Tocqueville), ook als dit apolitiek is. Maar de oplossing ligt eveneens niet bij deze instituten want ook hierin zijn de rijkeren oververtegenwoordigd (3x). De instituten die het traditioneel opnemen voor de onderste klasse, bv. vakbonden moeten dan weer inboeten tegenover de postmateriële waarden (bv. sluiting fabriek want milieu). We moeten dus opzoek gaan naar groepen die deze stratificatie kunnen tegengaan. We denken dan in de eerste plaats aan de partijen. De meeste, vooral langs de linkerkant, proberen zo namelijk iedereen bij eenzelfde verhaal te betrekken. Voor de Amerikaanse partijen is dit lastiger omdat zij sterk gefragmenteerd zijn. We zien dat bij al diegenen die gerekruteerd werden de Republikeinen de rijkste rekruten hebben en dat wie gevraagd wordt om mee te werken aan een campagne of te doneren ver boven het gemiddelde inkomensniveau vallen. Dus hoewel partijen hun best doen alle lagen van de maatschappij te rekruteren om te gaan stemmen, lukt dit niet bij hun rekrutering.

What about Unions? What about Churches?Amerikaanse vakbonden staan doorgaans erg zwak en zijn niet gelieerd aan partijen. De Kerk daarentegen staat erg sterk in de Amerikaanse samenleving en kan alle lagen van de bevolking raken (zie fig. 12-7 p. 455). Bovendien gaan de leden van een kerk ook veel vaker naar bijeenkomsten dan leden van een vakbond dat doen.

Participatory Equality and the Strucuture of Civil Society in America

De zwakte van de Amerikaanse vakbonden draagt niet bij aan de ongelijkheid aangezien de Kerk dit opvangt. De thema’s waarvoor de Kerk zijn leden mobiliseert zijn echter vaak niet die thema’s die de lagere klassen zou moeten aanbelangen (bv. abortus, euthanasie, …). In dat opzicht zijn ze dus een zwak alternatief voor vakbonden. Zolang geld het medium blijft om in de politiek te participeren, zal deze klassenkloof enkel maar toenemen.

27

Page 28: Facultair Overlegorgaan Sociale Wetenschappen … · Web viewSolidariteit op de werkvloer maakt deel uit van een ruimere sociale solidariteit 2) Kerk en religiositeit: Georganiseerde

DAAN DELESPAUL 2017-2018

12. S. Mariën, M. Hooghe & E. Quintlier (2010) : Inequalities in Non-institutionalised Forms of

Political Participation : A Multi-level Analysis of 25 Countries (p.187-213)

Verschillende studies tonen aan dat de klassieke participatie in West-Europa afneemt, maar de niet-geïnstitutionaliseerde vormen van participeren dan weer toenemen. De resultaten van deze studie wijzen uit dat deze nieuwe vormen de ongelijkheid op basis van onderwijs doen toenemen,

maar deze op basis van leeftijd en gender doen afnemen.

De traditionele massa-organisaties in het middenveld worden steeds meer omgevormd in professioneel gemanagede verenigingen, waar financiële bijdragen eerder dan vrijwilligerswerk de macht overnemen. Dit zorgt er mede voor dat de participatie in deze organisaties afneemt ten voordele van participatie door nieuwe media. De grens tussen een traditioneel en nieuw participatiemiddel is evenwel niet steeds duidelijk. Het onderscheidingscriterium wordt zo beter gelegd op mate van institutionalisering en niet op ‘oud’ en ‘nieuw’. Traditionele vormen zijn vaak nauw verworven in het electorale proces, dat wil zeggen direct optreden binnen de politiek, terwijl deze nieuwe vormen er liever voor kiezen invloed uit te oefenen van buitenaf. Dit leeft verder door in het idee van de ‘monitoring citizen’ die niet deelneemt aan het politieke proces tenzij nodig, maar van buitenaf de wacht houdt met interesse. Zij worden gezien als ‘everyday makers’ die elementen van politieke besluitvorming in hun dagelijks leven integreren zonder als zodanig zelf te participeren. Dit alles past in het plaatje van de postmaterialistische waarden.

The Problem of Inequality

Ideaal van de ‘inclusieve samenleving’ waar ieders belangen evenwaardig vertegenwoordigd worden. In de praktijk uit zich dit in het ‘one man, one vote’-principe, maar hierbij wordt geen rekening gehouden met de invloed van participatie. De wet stelt iedereen wel vrij om te participeren, maar in het participeren zelf blijkt een ongelijkheid schuil te gaan omdat het geld en kennis vereist. Bovendien blijkt ook dat de hogere klassen daarboven meer participeren. De vraag in dit artikel is of deze niet-geïnstitutionaliseerde vormen van politieke participatie mee kunnen helpen aan een gelijkere politieke maatschappij. Een eerste rooskleurig scenario is dat deze niet-geïnstitutionaliseerde vormen van politieke participatie inderdaad meer aanspreken aan een bredere laag van de maatschappij doordat zij simpeler in uitvoering zijn en minder tijd vragen. Een tweede hypothese gaat ervan uit dat er geen verschil zal komen, aangezien dezelfde personen blijven participeren, ongeacht het medium. De derde hypothese gelooft in een participatieparadox: de nieuwe vormen van participatie zijn juist ingewikkelder en vragen meer tijd, waardoor de ongelijkheid juist toeneemt.

Sources of Inequality

- Opleidingsniveau: hoger opgeleiden hebben meer civic skills, zijn eerder geneigd te participeren en gevraagd te worden te participeren en verdienen meer. Het internet wordt gezien als het exclusieve domein voor hoger opgeleiden.

- Gender: maar niet-geïnstitutionaliseerde vormen van politieke participatie zijn attractiever voor vrouwen omdat ze buiten de politiek plaatsvindt waar er eerder een mannelijke dominantie is (bv. in partijen). Ten tweede staan deze vormen dichter bij het alledaagse leven en vragen ze minder tijd.

- Leeftijd: vooral jongeren actiever op internet.

Presentation of the Data

Stemmen blijkt een vrij democratische vorm van participatie, tussen de opleidingscategorieën zit maar tien procent en qua gender zien we een gelijke verdeling, maar voor leeftijd blijkt dat jongeren veel minder gaan stemmen. Bij de resultaten voor gender zien voor over het algemeen weinig verschil, een aantal vormen van participatie krijgen wel de voorkeur bij vrouwen, andere dan weer bij mannen. Een groot verschil blijkt wel in leeftijd, jongeren zijn veel sneller weg met de niet-geïnstitutionaliseerde vormen van politieke participatie, de ouderen dan weer met traditionele participatie.

Results of the Multi-Level Model

28

Page 29: Facultair Overlegorgaan Sociale Wetenschappen … · Web viewSolidariteit op de werkvloer maakt deel uit van een ruimere sociale solidariteit 2) Kerk en religiositeit: Georganiseerde

DAAN DELESPAUL 2017-2018

Twee tabellen p.199-201: voor geïnstitutionaliseerde vormen van politieke participatie hebben mannen de overhand, bij niet-geïnstitutionaliseerde vormen van politieke participatie is dat effect omgekeerd. Ditzelfde geldt voor leeftijd waarbij de jongeren eerder voor de nieuwe vormen kiezen. Voor opleidingsniveau blijkt er geen noemenswaardig verschil te zijn tussen de tabellen, dit geldt ook voor bnp per capita. De mate van democratie maakt wel een verschil uit: niet-geïnstitutionaliseerde vormen van politieke participatie zijn populairder in democratische staten.

Further Analysis

Tabel 7 neemt een extra interactie-effect tussen Freedom House-index en de individuele attitudes mee, maar dit heeft weinig invloed. Educatie en politieke kennis blijven zo doorslaggevend voor het onderscheid. Uit analyse van odds-ratio’s blijkt dat politieke interesse een doorslaggevendere rol heeft.

Conclusie

Niet-geïnstitutionaliseerde vormen van politieke participatie betrekken vrouwen meer bij de politiek dan de traditionele vormen. Dit is een interessante bevinding die uitwijst dat het verschil in participatie niet zozeer bij een verschil in interesse valt te zoeken. Om de onderrespresentatie van vrouwen in de poltitiek aan te pakken moeten er dus institutionele hervormingen worden doorgevoerd. Hetzelfde kan gezegd worden voor jongeren. Dit lijkt dus ons ideaal van de inclusieve democratie te benaderen. Hogeropgeleiden, die al meer in de traditionele vormen participeerde, doen dit echter nog meer in de nieuwe vormen. Het is echter onmogelijk om te bepalen of het verschil bij diploma, dan wel inkomen ligt. De niet-geïnstitutionaliseerde vormen van politieke participatie zijn daarom geen goed medium om lageropgeleiden bij de politiek te betrekken, in tegendeel zelfs: het zorgt voor een vervreemding en kan tot extreem stemgedrag leiden.

13. R. Putnam (2007) : E Pluribus Unum : Diversity and Community in the Twenty-first

Century (p.137-174) Etnische diversiteit neemt toe in de meeste ontwikkelde landen door immigratie. Ondanks de voordelen op lange termijn heeft het invloed op de sociale diversiteit en het sociaal kapitaal.

Vertrouwen, altruïsme en buurtsamenwerking nemen erdoor af. Bepaalde multiculturele samenlevingen hebben dit echter kunnen overwinnen door een nieuwe vorm van sociale

solidariteit en identiteit te creëren.

Sociaal kapitaal bestaat uit sociale netwerken en de bijbehorende normen van wederkerigheid en vertrouwen. Dit heeft een invloed op het individuele niveau (bv. in het vinden van een job, voor gezondheid, …), maar ook voor de samenleving in het algemeen. Zo blijkt bijvoorbeeld dat de criminaliteit erdoor kan dalen. In het artikel bekijken we de invloed van immigratie en diversiteit op dit sociaal kapitaal.

The Prospects and Benefits of Immigration and Etnic Diversity

Immigratie is over de jaren heen toegenomen. Dit heeft vele voordelen: de creativiteit neemt toen (veel wetenschappers en artiesten van andere origine + intellectuele diversiteit), netto economische groei (hoewel discutabel voor de laagste klassen), kan helpen bij de baby-boom en zou bijdragen aan de economie van de vertreklanden ondanks de braindrain.

Immigration and Diversity Foster Social Isolation

Er zijn twee belangrijke theorieën omtrent diversiteit en sociale interactie. De eerste, de contacthypothese, stelt dat diversiteit de interetnische tolerantie en sociale solidariteit doet toenemen. Door contact zou immers het vertrouwen toenemen. Verschillende studies onderbouwen deze opvatting. De conflicttheorie wijst op nadelen voor groepsgevoel, sociaal vertrouwen, investering in publieke goederen, vrijwilligerswerk en zelfs carpooling. Om deze twee theorieën te vergelijken moet men rekening houden met twee belangrijke concepten: ‘bonding’ en ‘bridging’ sociaal kapitaal. Bonding kapitaal gaat om interactie met mensen zoals jezelf: zelfde leeftijd, gender, achtergrond, klasse, …, bridging gaat dan weer over die grenzen heen. De

29

Page 30: Facultair Overlegorgaan Sociale Wetenschappen … · Web viewSolidariteit op de werkvloer maakt deel uit van een ruimere sociale solidariteit 2) Kerk en religiositeit: Georganiseerde

DAAN DELESPAUL 2017-2018

contacttheorie stelt dat diversiteit de out-groupsolidariteit of bridging sociaal kapitaal laat toenemen, terwijl de conflicttheorie de in-groupsolidariteit of bonding sociaal kapitaal laat toenemen (etnocentrisme). Het probleem is dat onderzoeken enkel de visie naar de out-group meten en niet de relatie met de in-group, maar deze variëren onafhankelijk. Zo zou het kunnen dat diversiteit voor zowel de in- als out-groep positief/negatief is. Dit is de ‘constrict theory’. In deze analyse worden individuele attitudes vergeleken met sociaal kapitaal binnen een gemeenschap. Een eerste vaststelling is dat het interraciale vertrouwen (dus vertrouwen in andere rassen) hoger is in homogene gemeenschappen, dan heterogene (fig. 3, p. 147). Dit lijkt de conflicttheorie te bevestigen, maar het verhaal is veel ingewikkelder. Een tweede vraag richt zich op het vertrouwen van de respondenten in de inwoners van hun eigen buurt (en dit is in de VSA, dus veel buurten zijn raciaal gesegregeerd). Hier zien we een bijna identiek patroon (fig. 4, p. 148). Daarna vraagt men in hoeverre men personen van hetzelfde ras vertrouwt (fig. 5, p. 149). Hieruit blijkt dat het vertrouwen binnen de in-group ook lager is in heterogene gemeenschappen, wat dan weer in strijd is met de conflicttheorie. Met andere woorden: in een diverse samenleving is het vertrouwen in zowel de in- als out-groep lager. De laatste grafiek op p. 150 toont het ‘etnocentrisch vertrouwen’ (vertrouwen in eigen ras-interraciaal vertrouwen) waaruit blijkt dat er compleet geen correlatie tussen vertrouwen en diversiteit is waardoor noch de conflicttheorie noch de contacttheorie aan de sociale realiteit van Amerika beantwoorden. Het bevestigt echter wel de constrict theory, namelijk dat diversiteit leidt tot sociale isolatie. In gemeenschappen met veel diversiteit blijkt er een kleiner vertrouwen in de lokale overheid en media en invloed daarop, lagere opkomst, maar wel meer politieke interesse, kennis en participatie in niet-geïnstitutionaliseerde vormen, minder buurtsamenwerking, minder vrijwilligerswerk, lagere levenskwaliteit, minder vriendschappen en meer tijd achter de televisie. Diversiteit zorgt niet voor een conflict, maar zorgt er eerder voor dat men zich terugtrekt uit het gemeenschapsleven. Daar stuurt men aan op sociale hervormingen zonder de tijd te nemen deze te verwezenlijken. Twee zaken moeten hierbij worden opgemerkt. Ten eerste werd tot nu toe de gemeenschap als meeteenheid gebruikt, maar werd er niet gekeken naar uit wie die gemeenschap precies bestaat op individueel vlak en wat we over een persoon kunnen zeggen naarmate de buurt waarin hij woont. Ten tweede moeten we ook naar aspecten van een buurt kijken die losstaan van diversiteit, bijvoorbeeld criminaliteit, inkomen, etc. In de multivariate analyse blijkt dat de individuele variabelen (waaronder etniciteit) het sterkste effect geven, daarna contextuele variabelen. Daaruit blijkt inderdaad dat diversiteit een invloed heeft op het vertrouwen, ondanks de andere omstandigheden. Hierbij moeten nog enkele kanttekeningen worden gemaakt. Ten eerste zou het kunnen dat juist tolerante mensen diverse buurten kiezen, of andersom weinig actieve mensen naar deze buurten trekken (zelfselectie). Ten tweede is men de interactie-effecten niet nagegaan. Deze zouden weinig verschil maken, behalve dan dat in Amerikaanse context er een verschil tussen ‘Afro-Amerikaan’ en ‘van vreemd origine’ bestaat. Binnen de leeftijdscohorten is eveneens geen verschil, maar de jongsten zijn toleranter. Ten derde is de definiëring van ‘buurt’ erg moeilijk: waaruit bestaat iemands directe omgeving? In New York zijn er bijvoorbeeld veel verschillende buurten in één ‘buurt’ en men zou zich kunnen afvragen in hoeverre een inwoner van Manhattan in dezelfde buurt als een inwoner van The Bronx leeft. De hypothese is dat het effect van diversiteit enkel zou toenemen als we de buurten ‘verkleinen’, met weinig effect voor dit onderzoek. Ten vierde blijft de vraag wat de invloed van economische diversiteit zou zijn. In de tabellen bleek deze vrij parallel te lopen met deze van diversiteit, bovendien zou dit het isolationisme ook kunnen doen toenemen. Deze analyse zou echter te ver leiden aangezien het effect niet-lineair en bovendien interactief naar county zou zijn. Meest fundamenteel hierin is echter dat het effect niet significant is en daarom de relatie diversiteit-sociaal kapitaal niet kan verduisteren. Ten vijfde snap ik p.157-158 niet.Belangrijk blijft bovendien dat deze vorm van analyse geen informatie biedt over de dynamiek van een samenleving, daarvoor is cross-temporeel onderzoek vereist.

Becoming Comfortable with Diversity

Sociale psychologen hebben uitgewezen dat samenwerking makkelijker is naarmate de sociale afstand verkleint. Dit geeft het gevoel van een gedeelde identiteit, terwijl wanneer deze groot is men de ander als behorend tot een andere categorie gaat zien. De link tussen identiteit en sociaal kapitaal is dus snel gelegd. Sociale identiteit is echter een maakbaar gegeven. Vroeger zou men bijvoorbeeld groepen eerder indelen op basis van religie terwijl dat nu misschien veeleer door afkomst gebeurt. Met andere woorden het bridging kapitaal voor andere religieuze groepen is toegenomen (zonder daarom per se het bonding kapitaal met de eigen religieuze groep te verwaarlozen). De identiteitslijn kan daarom perfect elders worden getrokken dan op etniciteit. Dit is bijvoorbeeld gelukt in het leger van de VSA. Bovendien nam de VSA veel tradities van andere culturen toch geruisloos over (bv. pizza, Saint Patricksday, …). Ook interraciale

30

Page 31: Facultair Overlegorgaan Sociale Wetenschappen … · Web viewSolidariteit op de werkvloer maakt deel uit van een ruimere sociale solidariteit 2) Kerk en religiositeit: Georganiseerde

DAAN DELESPAUL 2017-2018

huwelijken nemen toe. De Amerikaanse identiteit is historisch gezien dan ook erg dynamisch. De uitdaging is dus niet om de immigranten te amerikaniseren, maar een nieuwe definitie van ‘wij’ te creëren over de etnische grenzen heen. Putnam stelt enkele beleidslijnen voor:

- Meer interactie bevorderen in scholen, community centres, sportclubs, …;- Staatssteun uitbreiden voor immigranten die Engels leren en ook hier integratie en

groepsvorming bevorderen;- De voordelen op nationale schaal (bv. economie) laten doorwegen in de nadelen op lokaal

niveau;- Lokale programma’s om immigranten de hand uit te reiken met een focus op wederzijds

leren. Meer bridging kapitaal tussen etnische organisaties en bv. kerken (bonding met bridging kapitaal laten samengaan)

14. M. Hooghe, T. Reeskens, D. Stolle & A. Trappers (2009): Ethnic Diversity and Generalized

Trust in Europe (p.198-223) Onderzoeken die een negatieve relatie tussen etnische diversiteit en algemeen vertrouwen

vaststellen zijn vaak enkel op Noord-Amerika georiënteerd. Deze studie richt zich daarom op Europa. De relaties op individueel vlak blijken dezelfde, maar op de pessimistische visie van

Putnam ziet men niet terug in een studie van de landen.

Research on Diversity and Generalized Trust.

Omdat een begrip als sociaal kapitaal vaak moeilijk te vatten is gebruikt men meestal de variabele ‘algemeen vertrouwen’. Verschillende onderzoeken wezen uit dat diversiteit een invloed heeft hierop, net als op de mate van isolatie en zelfs geluksgevoel. Uit aanvullend onderzoek blijkt dat socio-economische omstandigheden en mate van vertrouwen eveneens samenhangen, dit kan daarom als een controlevariabele gelden.

Three Advances in Research on the Effects of Diversity

De auteurs stellen drie aanvullingen voor aan het onderzoek. Ten eerste moet het onderzoek worden doorgetrokken naar andere landen alvorens we een uitspraak kunnen doen over Westerse democratieën. De VSA kende immers een heel andere migratiegeschiedenis en de inkomenskloof is er groter (dit is eveneens een voorspellende factor voor algemeen vertrouwen). Ten tweede wil men een andere indicator voor diversiteit gebruiken dan via percentages. Er is immers een verschil tussen migrantengroepen die al lang in een land verblijven en diegenen die er pas zijn; het is een dynamisch proces. Van de groep die reeds lang in een land verblijft kan niet verwacht worden dat ze dezelfde invloed op het vertrouwen heeft als een nieuwe inwoners die bijvoorbeeld door een internationale crisis in snel tempo toenemen. Bovendien zijn er ook verschillende vormen van immigratie (bv. arbeidsmigratie) en moet er ook rekening gehouden worden met de culturele afstand tussen de allochtoon en de ontvangende samenleving. Dit onderzoek neemt een statische term (percentage), een dynamische en de mate van afstand op. Ten derde moet men van een single-level analysis naar een multi-level analysis gaan. Diversiteit is immers een contextueel fenomeen.

Data and Method

Een ideale multi-levelanalyse vereist tenminste 30 cases, deze telt er maar 20 waardoor ze vatbaar voor outliers wordt (bv. Zwitserland voor aandeel buitenlandse populatie).

Results

De determinantiecoëfficiënt bedraagt 67% voor een onderzoek op basis van landen, hetgeen verklaard waarom een single-level analysis onvoldoende is. Zie tabel 1, p. 210: vrouwen, ouderen, rijkeren, hogeropgeleiden en kerkgangers hebben meer vertrouwen. Op het landenniveau zien we een sterk effect voor bnp en inkomensongelijkheid. In het multi-level model voor statische diversiteitsvariabelen valt op dat diversiteit geen invloed heeft op het algemene vertrouwen. Het aantal buitenlanders binnen een land zou dat wel doen wanneer we Zwitserland uit de analyse halen. In het multi-level model voor dynamische diversiteitsvariabelen komt een negatief effect voor vertrouwen naar boven in vertrouwen bij een sterke stijging van arbeidsmigratie, maar deze valt

31

Page 32: Facultair Overlegorgaan Sociale Wetenschappen … · Web viewSolidariteit op de werkvloer maakt deel uit van een ruimere sociale solidariteit 2) Kerk en religiositeit: Georganiseerde

DAAN DELESPAUL 2017-2018

grotendeels te verklaren door outlier Italië. Voor de rest kunnen we geen statistisch effect waarnemen.Tenslotte blijkt ook geen effect voor aantal goedgekeurde asielaanvragen.

Discussion

Op individueel niveau kwamen dezelfde resultaten naar boven: mannen, ouderen, laagopgeleiden en werklozen hebben minder vertrouwen. Op het landelijke niveau zien we echter dat diversiteit en de influx van vreemdelingen geen effect heeft op het vertrouwen van een samenleving. De kleine populatiegrootte kan er echter voor zorgen dat sommige effecten onderschat worden. Een ander probleem is het aggregatieniveau: een landelijk niveau is mogelijks te hoog om verschillen in sociale interactie binnen buurten te vatten.

15. S. Dion (2017): La diversité est un fait, l’inclusion un choix. (p.1-11)

In een toespraak legt de Canadese ambassadeur de immigratie- en integratiepolitiek van zijn land uit. Een essentiële factor is de Canadese ‘welkomscultuur’.

Canada heeft een erg diverse maatschappij met rond de 40% van de populatie met een immigratieachtergrond. De bevolkingsstijging van de OESO-landen hangt enkel af van immigratie aangezien de oorspronkelijke bevolking blijft verouderen. Bovendien heeft onze industrie deze mensen nodig. Er zijn met andere woorden redenen voor de “Wir schaffen das”-politiek van Merkel en Trudeau.

Canada en België zijn erg gelijkaardig: beiden diverse, tweetalige en federale staten. Canada stond initieel niet open voor immigratie, niet uit vrees voor het andere, maar omdat men geloofde in een uniforme staat. Dit zou het beleid, de sociale cohesie en de gelijkheid ten goede komen. In de 19de eeuw begon men daarom met een assimilatiepolitiek ten aanzien van minderheden (in het bijzonder de Franstaligen). Men had op zich niets tegen Franstaligen, maar meende dat het beter voor henzelf was om “Brits” te worden zoals de anderen. Gelukkig werd deze weg niet verder bewandeld, er zijn immers veel betere systemen van integratie. Het Canadese puntensysteem selecteert immigranten op basis van concrete noden. Dit wordt gezien als een maatschappelijke filosofie van multiculturalisme, die de hele bevolking raakt.

Het grote voordeel van Canada blijft zijn geografie. In tegenstelling tot Europa kunnen migranten niet gemakkelijk naar Canada gaan, dit zorgt ervoor dat zij hun selectieprocedure dan ook effectief kunnen doorvoeren. Bovendien wordt ook de integratie ermee verbetert, omdat men psychologisch verder van het thuisland staat. Nog een voordeel is dat men kan rekruteren uit een veel groter gebied en niet enkel een regio en daarbij die migranten kan selecteren die hoogopgeleid zijn. Dit alles draagt bij aan de welkomscultuur in Canada.

Toen president Trudeau aankondigde 25000 Syrische vluchtelingen op te nemen werd dit enthousiast onthaald. Omdat de bevolking wist dat dit een vast cijfer was dat waarschijnlijk niet opeens kon stijgen, kon dit de xenofobische reflexen inperken. Dit staat tegenover Duitsland, waar de bevolking na de verwelkomingspolitiek van Merkel vreesde voor een tsunami. Een van de drijfveren voor de Brexit was zo bijvoorbeeld “de vrees de controle over de eigen grenzen te verliezen”. Na een recente verandering van de Amerikaanse politiek staan nu ook vluchtelingen aan de Canadese grenzen, wat ook opeens bij hen voor ophef zorgt. Trudeau meent dat dit de Canadezen daarom meer vertrouwen hebben en meer verwelkomend zijn wanneer er een duidelijk, integer immigratiesysteem is.

Ook het Canadese integratiesysteem is bewonderenswaardig. Volgens OESO-criteria presteert het land bovengemiddeld op alle variabelen. Het feit dat Canada “maar” zesdes op een of andere integratieranking staat is te wijten aan de beperkte toegang voor sans-papiers tot gezondheidszorg, doordat vreemdelingen niet kunnen stemmen en iets met tijdelijke jobs, maar dat gaan ze nu oplossen!! De grote voordelen zijn dan weer de snelle en simpele procedure naar het burgerschap, de snelle toegang tot de arbeidsmarkt en de steun van de regering aan nieuwkomers, bv. in de vorm van taallessen en onderwijs. Het scoort ook een eerste plaats in zijn antidiscriminatiepolitiek en intercultureel overleg. De grootste factor is echter de volksfilosofie van multiculturalisme die in het hart van alle Canadezen wordt gedragen. (in Québec zeggen ze liever interculturalisme omdat de migranten nog Frans moeten leren of zoiets)

32

Page 33: Facultair Overlegorgaan Sociale Wetenschappen … · Web viewSolidariteit op de werkvloer maakt deel uit van een ruimere sociale solidariteit 2) Kerk en religiositeit: Georganiseerde

DAAN DELESPAUL 2017-2018

Aan de nieuwkomers wordt niets opgedrongen en ze worden actie aangespoord hun eigen cultuur te delen. Dit laat de immigranten ook trots voelen over hun nieuwe land en zorgt ervoor dat ze eerder geneigd zijn haar waarden te accepteren. Dubbele paspoorten zijn ook essentieel om aan te tonen dat men zijn oude waarden niet hoeft op te geven.

16. E. Uslaner (2017): The Historical Roots of Corruption (p.1-32)

Inleidende tekst over het ontstaan en wegwerken van corruptie. Essentieel is onderwijs, dat gelijkheid vergroot en civic values promoot.

Griekenland kent veel corruptie en dat verhoogt de ongelijkheid. De crisis zou te wijten zijn aan een erg logge en inefficiënt overheidssysteem, maar ook aan een gesloten economie in handen van enkele families die door de politiek gesteund worden. Corruptie is iets dat moeilijk kan weggewerkt worden omdat het diep in het sociale leven en de politieke geschiedenis aanwezig is. Landen met meer gelijkheid en die meer diensten aan hun bevolking verschaffen zijn het minst vatbaar voor corruptie. Een van die diensten, onderwijs, zorgt voor een afname van corruptie in de toekomst.

Onderwijs zorgt voor een gelijkheid en leert haar burgers ‘civic values’ (=ook identiteit) waardoor zij zelf in de politiek kunnen gaan. Bovendien promoot het een loyaliteit aan de staat omdat men er gemakkelijk de voordelen van kan zien. Hierin speel path dependency een belangrijke rol: landen met goed onderwijs zullen dit in de toekomst ook hebben, landen zonder corruptie hebben dit in de toekomst ook. Landen met een lange traditie van ongelijkheid (bv. ex-koloniale landen), hebben dit zo doorgaans nu nog steeds en met zich veel corruptie. Uit onderzoek blijkt dat landen met goed/veel onderwijs in 1870 de minst corrupte zijn in 2010, het effect van onderwijs op ongelijkheid is dus blijvend over 140 jaar.

De meeste literatuur zoekt naar manieren om corruptie op korte termijn weg te werken. Met kleine hervormingen en een betere dienstverlening voor basisartikelen zou een basis kunnen gelegd worden voor institutievorming en vertrouwen van de bevolking, wat een goede start zou zijn. Andere onderzoeken leggen de focus op transparantie, evaluatie van overheidsprogramma’s en meer interne competitie voor benoemingen. Maar corruptie lijkt nog steeds te bloeien, 131 van 178 landen zijn volgens Transparency International nog steeds ‘corrupt’. De kortetermijnoplossingen zijn immers niet rendabel op lange termijn.

De roots van corruptie liggen in de geschiedenis van een land. Hiervoor zijn drie factoren van belang: een vroege democratisering vermindert de kans, sterke economische context (bv. veel mineralen, vruchtbaar land, maritiem, …) wat een langetermijneffect op de economie heeft en nieuwe technologie. Gewassen als graan en tarwe konden op onafhankelijke boerderijen worden gekweekt, luxegoederen als thee en tabacco en mijnen vereisten dan weer slaven, wat bijdroeg aan de ongelijkheid. Landen met malaria waren dan weer onderontwikkeld. Andere theorieën focussen op de mate van technologie na 1500, wat een blijvende invloed heeft op de economie van vandaag.

The Roots of Corruption

Corruptie maakt deel uit van een vicieuze cirkel (“inequality trap”):

ongelijkheid laag vertrouwen corruptie meer ongelijkheid

De armen hebben de macht niet om deze cirkel te doorbreken. Ongelijkheid draagt dan ook bij aan corruptie omdat: 1) de gewone burgers geloof in het systeem verliezen, 2) het idee krijgen dat ze toch niets kunnen veranderen en daarom zelfs zullen meewerken en 3) de instituten van de overheid door corruptie geen legitimiteit meer hebben. Corruptie leidt dan weer tot een verlies in algemeen vertrouwen, maar dit geldt enkel voor grote corruptie. Dit laag vertrouwen leidt dan weer naar de ongelijkheid en dan is de cirkel rond. Met andere woorden: ongelijkheid, laag vertrouwen en corruptie blijven vast omdat ze in een vicieuze cirkel zitten.

Naast de inequality trap is er ook een “corruption trap”. Door corruptie kunnen de landen ook geen belastingen ophalen om in onderwijs te voorzien. Burgers zullen bovendien een belastingverhoging nooit vertrouwen om deze reden omdat ze er niet in geloven dat het

33

Page 34: Facultair Overlegorgaan Sociale Wetenschappen … · Web viewSolidariteit op de werkvloer maakt deel uit van een ruimere sociale solidariteit 2) Kerk en religiositeit: Georganiseerde

DAAN DELESPAUL 2017-2018

programma zal werken. De overheid kan echter niet meer vertrouwen wekken zolang ze geen financiële middelen hiervoor heeft.

In de VSA was er vooral de Progressive movement die ijverde voor een zuiverdere overheid en meer gelijkheid. Dit moest gebeuren door onderwijs aan iedereen aan te bieden naar Scandinavisch en Duits model (Wisconsin kende dan ook veel Scandinaviërs). Dit noemt men het “Wisconsin Idea”.

Conclusie: onderwijs en corruptie hangen samen, onderwijs doet het algemeen vertrouwen toenemen en onderwijs zorgt voor economische groei en meer economische gelijkheid.

Corruption and Education

1. Onderwijs zorgt voor een brede set van gedeelde waarden en meer sociale banden2. Onderwijs zorgt voor een sterke groei van economische gelijkheid, waardoor de

huidige elite kan worden uitgedaagd3. Onderwijs versterkt economische groei en verrijkt de armen, waardoor die zich niet

meer op cliëntalisme moeten beroepen4. Onderwijs bevordert de gendergelijkheid5. De pers heeft een belangrijke taak in het aankaarten van corruptie, zonder

geletterdheid is de pers dode letter

Alternative Explanations for Corruption

Het kan zijn dat historische factoren het heden bepalen, niet het beleid. Waarden worden vooral gedragen door een bevolking en niet opgelegd, de landen die het nu slechter doen zijn vaak die ex-kolonies die niet bevolkt werden door een nieuwe bevolking. Een van de belangrijkste historische factoren is de werking van de instituten. Nog een historische reden kan liggen in het functioneren van de partijen. Een onderscheid wordt gemaakt tussen programmatische partijen die op zoek gaan naar een brede steun voor hun beleid, andere partijen zijn particularistische partijen waar de corruptie hoger ligt.

Ook klimaat is een determinerende factor voor de economie. Geografie en klimaat dragen tussen 40% en 50% bij aan de huidige economische situatie voor het westen. In die kolonies waar het beter leven was kwamen dan ook de meeste westerlingen, terwijl ze slaven naar de moeilijker te bewonen gebieden brachten. Het percentage Europeanen in een kolonie bepaalt de graad van onderwijs later. Let op dat klimaat ook niet alles bepaalt, kijk naar Noord- en Zuid-Korea

Historische factoren en instituten zijn geen reden om de algemene onderwijskwaliteit te bepalen, maar goede instituten komen wel voort uit onderwijskwaliteit. Katholicisme of Protestantisme bepalen evenwel de geletterdheid en wiskundekennis (protestantisme stond veel opener). Ook het klimaat bepaalt niet of er onderwijs is of niet, dat bepalen politieke besluiten, meer bepaald van de rijke landeigenaren die hun arbeiders niet wilde verliezen.

Grotere economische gelijkheid zette het publiek aan algemene educatie te vragen, als er gelijkheid was zouden deze eisen niet per se tegen die van de elite moeten zijn. Koloniale machten hadden geen zin om de lokale bevolking van scholing te voorzien. Educatie was een politieke beslissing en niet afhankelijk van context of democratie.

17. M. Bovens en A. Wille (2017): Diploma Democracy (p.11-108 & 139-187)

PART I. Concepts and Contexts

HOOFDSTUK 2 – Diplomas

Meritocracy: From Science Fiction to Factual Description

Meritocratie = samenleving waar sociale status wordt bepaald door ‘bekwaamheden’ (merites).

34

Page 35: Facultair Overlegorgaan Sociale Wetenschappen … · Web viewSolidariteit op de werkvloer maakt deel uit van een ruimere sociale solidariteit 2) Kerk en religiositeit: Georganiseerde

DAAN DELESPAUL 2017-2018

Educatiegraad in 20ste eeuw nam fel toe, wat voor een nieuwe sociale stratificatie leidde. Diploma’s bepalen iemands kansen qua jobs (bv. Ellen Jonckheere, CEO van NMBS die geen Masterdiploma had en daarom niet verkozen werd). De langer de meritocratie bestaat des te groter de kloof, neem nu bijvoorbeeld Frankrijk met zijn Grand Écoles. De sociale mobiliteit in het algemeen neemt ook af op basis van onderwijs.

Meritocracy and Diploma

Een meritocratie wil carrière openlaten aan talenten ongeacht hun sociale status, maar wel op basis van verdiensten. Om te bepalen of een samenleving naar een meritocratie neigt kan men de OED-driehoek gebruiken (zie ppt.). Onderwijs is de motor van de meritocratie. Onderwijs verschilt echter van land tot land; om het te meten gebruikt men daarom internationaal de ISCED-schaal in drie niveaus: laag, gemiddeld en hoog (<10; 10<X<14; >14).

The Expansion of the Educational System

Stijging in aantallen, maar ook in diversiteit. Vanaf jaren zestig neemt de toegang tot hoger onderwijs in de meeste OESO-landen toe. Omdat de groep universitairen te groot werd ontstonden de hogescholen.

The Educational Revolution as a Critical Juncture

Parsons spreekt van de ‘onderwijsrevolutie’: een democratisering van het hoger onderwijs die voor een grotere gelijkheid van burgers zorgde. Deze uitbreiding, samen met de secularisering, zouden sociale mobiliteit in de hand werken en een van de factoren zijn in de evolutie van materiële naar post-materiële waarden van Inglehart. Allardt en Kriesi gaan nog verder en zien dit als een kritiek kruispunt betreffende nieuwe politieke verschillen en conflicten. Niet alleen zorgt een breuklijn op basis van onderwijs voor een sociale breuklijn, maar ook qua politieke attitudes maakt ze een verschil uit. De rol van onderwijs is daarom ambigu in de literatuur: sommigen menen dat ze de deur naar sociale mobiliteit openzetten, anderen dat ze juist nieuwe breuklijnen creëert en deuren sluit.

New Openings and Emerging Social Opportunities

Meritocratie is de nieuwe sociale ideologie in de post-industriële samenleving; gender, etniciteit en sociale achtergrond maken niet langer uit. De politiek heeft daarom geprobeerd onderwijs te democratiseren om iedereen dezelfde kansen te geven. Onderwijs zou sociale mobiliteit stimuleren.

New Closures and Diminishing Returns

Anderzijds biedt deze revolutie niet voor iedereen vooruitgang. Meritocratie zorgt ook voor sociale inclusie en een nieuwe hoogopgeleide aristocratie. Diploma’s werken nog te veel als ‘screening devices’ (bv. in het geval van Ellen Jonckheere) waarbij de sterkste jobs naar diegenen met een hoog diploma gaan, niet zozeer degenen met talent. Bourdieu ziet een diploma dan ook als een unieke vorm van sociaal kapitaal, waarmee de juiste kandidaten voor een positie worden uitgefilterd. Dit kapitaal zal echter afnemen naarmate de scholingsgraad toeneemt (‘credential inflation’). De hoogopgeleiden zijn echter geneigd om aan sociale reproductie te doen, waardoor dezelfde posities toch naar dezelfde personen zouden gaan. Onderwijs is dus helemaal geen neutrale rechter en door zelf een hiërarchische structuur aan te nemen (bachelor, master, doctoraat, …) promoot het een verdere stratificatie en ongelijkheid, niet het meritocratisch ideaal. Omdat onderwijs de factor van succes bepaald, worden jongeren veel vroeger geschift op de sociale ladder. Hoewel de eerste treden van het onderwijs parcours daarin insignificant zijn (middelbaar, lager onderwijs), is het universitaire parcours echter enorm belangrijk. Zeker de keuze voor een specifieke universiteit is daarin essentieel. Een diploma aan de KU Leuven wordt daarin nog steeds als hoger aanzien dan een van de UA. De mate van sociale mobiliteit en onderwijs is de laatste jaren, in plaat van toegenomen, juist afgenomen. Dit betekent dat de meritocratie in zekere mate afneemt ondanks de democratisering van onderwijs. Bovendien neemt het aantal ‘instromers’ in universiteiten (= van lage sociale klasse) af.

The Rise of the Well-Educated in Society

Op dit moment heeft een groter aandeel van de bevolking tertiaire opleiding gehad dan deze met primaire opleiding. Dit was doorheen de 20ste eeuw ondenkbaar. Dit maakt van de hogeropgeleiden echter nog geen meerderheid, maar een erg vocale en politiek actieve

35

Page 36: Facultair Overlegorgaan Sociale Wetenschappen … · Web viewSolidariteit op de werkvloer maakt deel uit van een ruimere sociale solidariteit 2) Kerk en religiositeit: Georganiseerde

DAAN DELESPAUL 2017-2018

minderheid. Aangezien dit ook zorgt voor een politieke breuklijn wordt dit een belangrijke uitdaging in de 21ste eeuw. De hogere participatiegraad van hogeropgeleiden stelt dan ook de democratie zelf in vraag.

HOOFDSTUK 3 – Democratie

Government by or for the People?

Een ‘diploma democratie’ is een democratie waarin de hoogopgeleiden de macht hebben. Is dit nog een democratie? Hierover bestaan twee zienswijzen: er is een democratie door iedereen en een democratie voor iedereen. De eerste versie vereist een maximale input van de hele bevolking in het maken van beleid. Deze vertoont het meeste gelijkenissen met de klassieke Atheense democratie, volgens dit idee is een diploma democratie dan ook te vermijden. Het probleem is dat deze vorm enkel in kleine staten, zoals Athene, zou werken. Voor grote natiestate werd het idee dan ook al snel vervangen door een representatieve democratie voor iedereen. In deze visie zijn burgers eerder controleurs dan participanten. Het idee daarnaast is dat de gemiddelde burger niet voldoende kwaliteiten heeft om beleid te voeren, maar kennelijk wel om te controleren. Vele grote politieke denkers hebben daarom ook meermaals hun vrees geuit voor deze tekortkoming van de democratie: brede participatie zou haar destabiliseren.

Principal Elements of Democracy

Representatie: handelen naar de belangen van de anderen, op een responsieve manier. De vraag is dan wie men vertegenwoordigt: de individuele persoon met zijn eigenschappen (bv. vrouwen, moslim, Vlaming) of het algemeen belang? Dit uit zich in het verschil tussen een mandataris en een trustee. Het idee bij deze mandatarissen zou zijn dat de politiek een correcte afspiegeling zou zijn van de samenleving. In dat opzicht is een diploma democratie dus een serieus probleem. Een perfecte representatie is echter nooit mogelijk: vele westerse democratieën ‘kiezen’ dan ook in zekere zin hun prioriteiten, in veel gevallen is dat een representativiteit voor vrouwen en minderheden. Moet dat ook gelden voor opleiding? Ook bij de trustee-visie (Burke, Mill) zijn er problemen: in hoeverre kan een trustee immers correct de laagopgeleiden vertegenwoordigen, zeker als er door een politieke breuklijn verschillen in visie zijn?Responsiviteit: het beleid van de politiek beantwoordt aan de noden van de bevolking. Ook deze kan zwaarder aangetast worden aan de noden van een hoogopgeleide bevolking. Verantwoording: functie van het parlement om de controle over de uitvoerende macht te houden. Hiervoor heeft zij enkel middelen zoals interpellaties, moties, vragen en plenaire debatten. Doordat beleid steeds complexer wordt blijft de vraag vaak wie verantwoordelijk is: de ambtenaren of een minister. Populaire media zijn dan ook vaak te ongenuanceerd in hun berichtgeving hieromtrent. Legitimiteit: de drie voorgaande begrippen helpen in de legitimiteit van een bestuur. Representatie voor de input, responsiviteit in de output en verantwoording in de throughput.

The Participatory Revolution

De tweede helft van de 20ste eeuw zag een modernisatie van de politieke instellingen. Dit ging gepaard met een uitbreiding van politieke participatie in de vorm van betogingen, petities en andere vormen van nieuwe participatie. Tegenwoordig hebben burgers veel meer middelen om de politiek op een correct niveau te volgen en te begrijpen. Dit heeft geleid tot nieuwe verwachtingen over de rol van burgers in politiek en het geloof dat zij zelf politieke beslissingen zouden kunnen maken. Bovendien wordt deze mening steeds vaker geuit. De overheid heeft hierop gereageerd met een meer horizontale manier van beleidsvoering waarbij burgers, middenveld en markt een uitgebreide inspraak krijgen in de totstandkoming van het beleid, met andere woorden wordt dus ook steeds meer participatie in de politiek toegelaten. Deze evolutie van government naar governance heeft de positie van de traditioneel representatieve instituten enkel maar verzwakt, maar de deur naar participatie verder geopend. De vraag blijft echter wie gebruik maakt van deze nieuwe participatiemedia.

New Openings and Emerging Political Opportunities

De participatierevolutie heeft de gelijkheid op het politieke veld drastisch verhoogd, vergelijkbaar met de onderwijsrevolutie dat voor de sociale klassen deed. Door een betere opleiding kregen inderdaad nieuwe groepen bevolking veel meer kansen in de politieke arena. Bovendien heeft het publiek een veel betere kans geïnformeerd te zijn door onder meer de expansie van het internet. Politieke partijen en organisaties spelen proberen deze interesse ook actief aan te wakkeren. Een identiteitspolitiek gericht op verschillende groepen van de bevolking heeft plaats gemaakt voor een engere individuele aanpak, waarbij wordt ingespeeld op persoonlijk relevante issues. Hierbij

36

Page 37: Facultair Overlegorgaan Sociale Wetenschappen … · Web viewSolidariteit op de werkvloer maakt deel uit van een ruimere sociale solidariteit 2) Kerk en religiositeit: Georganiseerde

DAAN DELESPAUL 2017-2018

duiken ook veel meer groepen op gefocust op een enkele zaak. Deze one-issuegroepen slagen er vaak in erg prominent aanwezig te zijn in het publieke debat en zo in te spelen op het systeem.De participatierevolutie biedt de politiek ook een veel ruimer beeld van de individuele wensen van haar burgers. Door verschillende polls en groepen wordt het duidelijk welk beleid de burger wilt. Ook de veelheid van verkiezingen draagt hieraan bij.

New Closure and Diminishing Democratization

Er is ook een duistere kant aan dit proces. Het politieke speelveld mag dan wel verder geopend zijn, dit neemt niet weg dat de kansen niet altijd even verdeeld zijn. Hoewel media steeds een duidelijker beeld kunnen scheppen van het politiek beleid zorgen de nieuwe informatiekanalen er vaak ook voor dat men zich kan afsluiten van de mainstream om zich te richten op de niche (media bias). Deze groep kan kiezen wat ze willen consumeren en ook enkel dat consumeren. Daarnaast is het tegenwoordig ook mogelijk om zich helemaal niets meer aan te trekken van politiek. Wie vroeger de krant las voor sportartikelen kan nu een gespecialiseerd tijdschrift nemen en zal daardoor niet meer in contact komen met andere artikelen. Verder blijven politieke partijen nog steeds de dominante actoren on het veld, maar hun ledenaantallen blijven toch dalen. Deze partijen geraken hierdoor steeds verder weg van de basis en het middenveld. De nieuwe civil-societygroepen kennen eenzelfde patroon. Na een initiële bottom-up groepsvorming neigen zij vaak naar een professionaliteit en specialisatie waardoor de originele link met de burger verdwijnt. Verder is er ook een probleem bij het uitbreiden van democratisering: dat van second-order elections. Deze worden vaak niet door een brede laag van de bevolking benut en verliezen daardoor aan legitimiteit. Nieuwe vormen van participatie liggen hoogopgeleiden nu eenmaal ook beter, omdat er vaak een focus op kennis wordt gelegd. Civic skills zijn bovendien vaak aangeleerd.

The Rise of the Well-Educated in Democratic Politics

De participatierevolutie is sterk gelieerd aan de onderwijsrevolutie. Niet alleen vonden ze plaats op hetzelfde ogenblik, bovendien versterken ze elkaar. Onderwijs geeft burgers de skills om aan het publieke debat deel te nemen en verkozen te worden in politieke organen. De horizontale besluitvorming geeft aan de burger ook de mogelijkheid zijn belangen aan het bestuur te kennen te geven. Vaak zijn het enkel de hoogopgeleiden die van deze mogelijkheid gebruik maken. De vraag blijft dan of deze verdere verdieping van de democratie niet gewoon een verschuiving naar een nieuwe elite inhoudt.

HOOFDSTUK 4 – Education as Cleavage

An Educational Cleavage?

Traditionele breuklijnen leven vandaag de dag door in een educatie-breuklijn. Tussen deze groepen bestaat er amper interactie, zo zijn 85% van de Nederlandse huwelijken tussen mensen van hetzelfde opleidingsniveau. Ook in de politiek is deze breuklijn te zien: hogeropgeleiden zullen vaker voor groene en sociaal-liberale partijen zoals GroenLinks en D66 stemmen terwijl lageropgeleiden eerder voor nationalistische en conservatieve partijen zullen kiezen. De auteurs willen dit effect verder nagaan in dit boek.

Cleavage as Social and Political Concept

De breuklijnentheorie werd als eerste opgesteld door Lipset en Rokkan in 1967. De sociaal-economische is de breuklijn die het langste leek te overleven. Door de jaren heen zijn deze breuklijnen evenwel afgezwakt doordat partijen meer op issues dan groepen spelen, anderen menen dat de focus van de breuklijnen gewoon simpelweg verschuiven. Recentelijk werd deze theorie nieuw leven ingeblazen door de vraag of opleiding zelf een breuklijn kan inhouden. Hierbij wordt scholingsgraad apart van inkomen onderzocht, vooral in relatie met post-materiële waardenvoorkeur.Een breuklijn bestaat uit drie factoren: een empirisch element dat we in sociaal-structurele termen kunnen vatten, een normatief element dat de identiteit en waarden van een bepaalde groep bepaalt en een organisatie– en gedragselement dat de interacties van de groepsleden bepaalt. Het bestaan van groepen met tegengestelde meningen volstaat niet, het is structureel ingebed en heeft een georganiseerde politieke expressie.Een breuklijn uit zich echter voornamelijk op individueel vlak, dit is dan ook het meetniveau dat we onderzoeken.

Social Differences: Educational Groups as Social Groups

37

Page 38: Facultair Overlegorgaan Sociale Wetenschappen … · Web viewSolidariteit op de werkvloer maakt deel uit van een ruimere sociale solidariteit 2) Kerk en religiositeit: Georganiseerde

DAAN DELESPAUL 2017-2018

De eerste vraag is of we structureel sociale verschillen kunnen zien tussen lageropgeleiden en hogeropgeleiden. We verwachten dit dan voornamelijk te zien in westerse landen, waar onderwijs voor iedereen toegankelijk is. Een breuklijn is vaak demografisch te zien, dat is ook hier het geval.Een tweede vraag is of deze demografische groepen zich uiten in sociale strata. Zoals eerder vermeld zijn jobkansen eerder toebedeeld aan hogeropgeleiden. Het gevolg hiervan is een grote inkomenskloof tussen de twee groepen.Ten derde is onderwijs ook een medium voor sociale stratificatie. Het blijkt immers dat huwelijken steeds vaker gebeuren tussen mensen met eenzelfde opleidingsniveau. De homogeniteit neemt daarbij zelfs toe naarmate het opleidingsniveau stijgt.

Attitudinal Differences: Cosmopolitans versus Nationalists

Opdat een sociale kloof politiek relevant wil worden moeten deze groepen verschillende waarden hebben. In welke mate hebben lageropgeleiden andere waarden dan hogeropgeleiden? Deze verschillende attitudes blijken in culturele problemen zoals immigratie en globalistische waarden zoals Europese eenmaking. Verschillen op de links-rechts schaal zijn minder prominent. Lageropgeleiden zijn wel eerder geneigd te stemmen op partijen die de inkomenskloof pogen te verminderen. Het verschil tussen de zogenaamde kosmopolieten en nationalisten blijkt dan weer veel duidelijker op educatie te zijn gestratificeerd. Het verschil is evenwel groter voor attitudes ten opzichte van immigratie dan ten opzichte van de EU. Ook tussen de landen zelf is er bovendien een groot verschil te merken., vooral Oost-Europa is nationalistischer.

Institutional Differences: Social-Liberal versus Nationalist Parties

Breuklijnen uiten zich in verschillend stemgedrag. Uit grafieken op p. 54-55 blijkt dat lageropgeleiden veel sneller op nationalistische partijen stemmen dan hogeropgeleiden, en het omgekeerde geldt voor groene en links-liberale partijen. Outliers aan nationalistische kant zijn Jobbik en Recht en Rechtvaardigheid. Hieruit blijkt ook dat sociale klasse en opleiding twee verschillende variabelen zijn, anders zouden de verschillen tussen links en rechtse partijen allicht groter zijn (zie vorige paragraaf). De traditionele socio-economische breuklijn bestaat evenwel nog, maar kan de opkomst van deze twee nieuwe politieke groepen niet verklaren. Op een vlak heeft de breuklijn zich evenwel nog niet ontwikkelt: namelijk dat de partijen in kwestie zich niet specifiek richten op de groepen die ze vertegenwoordigen (er bestaat bv. geen Universitair-Opgeleiden Partij). Het sociale stigma voor zulke partijen zou allicht groter zijn dan bij traditionele breuklijnen.

Different Stages of Cleavage Formation across Europe

Er is ten eerste en structureel element: over de laatste 100 jaar is de opleidingsgraad over Europa sterk toegenomen. Dit is een sterke factor in inkomensverschil en sociale homogeniteit, iets wat Lipset en Rokkan in gedachten hadden bij de definiëring van sociale breuklijnen. Segmentatie en segregatie is sterker te zien in het westen, stratificatie blijkt echter vooral zichtbaar in Oost-Europa. Ten tweede nestelt deze breuklijn zich verder op enkele kernwaarden en politieke voorkeuren. Hogeropgeleiden hebben eerder kosmopolistische waarden in verband met immigratie en Europese eenmaking. Dit is te zien in alle onderzochte landen. Deze waarden zorgden ook voor de oprichting van nieuwe politieke organisaties, die op hun beurt voor politieke breuklijnen zorgen. Deze organisaties zijn dan vooral de groene en nationalistische families van het politieke spectrum. De tabel op p. 58 toont de correlatie tussen de verschillende elementen van breuklijnen in relatie met het verschil in opleiding. Hieruit blijkt dat de vorming van deze breuklijn verschilt naargelang welk land we nemen. België is een van de landen waarin de verschillen het grootste zijn. Hierbij moet wel rekening gehouden worden met de specifieke politieke systemen en de geschiedenis van de landen in kwestie.

Cleavage formation and the Rise of Political Meritocracy

De breuklijn zal zich niet enkel uiten in politieke partijen, maar ook in andere aspecten van de samenleving. In het verdere onderzoek van dit boek worden 6 landen voor analyse geselecteerd omwille van hun brede scholingsgraad, naast het meritocratisch karakter van hun arbeidsmarkt en onderwijssysteem.

PART II. Contours

38

Page 39: Facultair Overlegorgaan Sociale Wetenschappen … · Web viewSolidariteit op de werkvloer maakt deel uit van een ruimere sociale solidariteit 2) Kerk en religiositeit: Georganiseerde

DAAN DELESPAUL 2017-2018

HOOFDSTUK 5 – Education Gap in Political Participation

The Hamburger Bildungskampf

Voorstel in Hamburg om onderwijskansen armen te verhogen door algemeen, uniform onderwijs met 2 jaar te verlengen. Hoogopgeleide ouders protesteerden hiertegen en wonnen uiteindelijk een referendum hiervoor. Het probleem was dat de opkomst hiervoor maar 12% was in arme buurten en tot 60% in rijke buurten. Dit is een voorbeeld van hoe het opleidingsniveau een rol in participatie kan spelen.

Education Gaps in Participation

Participatiepiramide: des te hoger des te moeilijker om te participeren (zie fig. 5.1 p.66). Het volgende hoofdstuk is op basis hiervan geordend.

I. Toeschouwerspolitiek

Dit is de meest rudimentaire vorm van politieke participatie: het gaat om het bekijken van politiek nieuws of bijvoorbeeld erover praten. De stijging aan informatie maakt ook dat het mogelijk is voor burgers herover te praten (‘monitoring citizen’). Het ESS heeft een aantal van deze aspecten onderzocht. Het kijken van politiek nieuws op tv verschilt nauwelijks voor de verschillende opleidingsgroepen, rond de 60% van de burgers over alle groepen kijkt dagelijks een half uur of meer. Wanneer men naar het lezen over politiek peilt, zien we wel een verschil: de hogeropgeleiden scoren daar veel hoger (rond de 30%). Tussen de lager– en middelopgeleiden is een kleinere kloof. Wat betreft het debatteren scoren de hoogopgeleiden opnieuw het hoogst rond de 70%, de kloof met de laagopgeleiden bedraagt daar 20%.

II. Stemmen

Het stemrecht is over de jaren uitgebreid en er zijn ook meer verkiezingen. Het is een vorm van participatie die weinig moeite vereist en bovendien eerlijk verdeeld is (‘one man, one vote’). Hoewel op individueel vlak opleidingsniveau een sterke indicator is om te gaan stemmen, is het dat op geaggregeerd niveau niet: de scholingsgraad nam in de 20ste eeuw enkel maar toe, terwijl de opkomst steeds lager werd/wordt. Brody noemde dit de ‘puzzle of political participation’. Dit kan echter verklaard worden doordat de participatiekloof toeneemt: lage opkomst gaat gepaard met een grote participatiekloof. Los van deze hypothese, toont de ESS-data een kloof in het gaan stemmen. Dit hangt evenwel ten dele vast aan het type verkiezing. ‘Second order’-verkiezingen hebben een grotere kloof en sowieso een lagere participatie. Ten tweede zijn er contextuele factoren. Des te makkelijker het registratiesysteem om te stemmen en des te minder keuzes tussen partijen, des te meer de lageropgeleiden geneigd zijn te gaan stemmen. De auteurs zijn voorstander van een stemplicht. De verdere democratisering door meer verkiezingen heeft de kloof dus enkel maar verhoogt. Complexiteit van het systeem maakt het bovendien vaak moeilijk wie waarvoor verantwoordelijk is (zie Wallonië).

III. Niet-electorale activiteiten- Conventionele vormen: manieren om politieke proces buiten verkiezingen om

te beïnvloeden, zonder te blokkeren (bv. badge dragen, politici contacteren, etc.). Hier bestaat inderdaad een duidelijke kloof. Geld doneren en in een organisatie meewerken zijn de populairste vormen, hier is de kloof ook groter.

- Protestparticipatie: demonstraties, petities, boycots en buycots, enzo. Initieel ging men ervan uit dat het ook hier vooral de hoogopgeleiden waren die hun stem lieten klinken, Verba, Schlozman en Brady bewezen echter dat dit niet het geval is voor straatprotesten. Recente onderzoeken tonen toch een kloofje, maar dit is maar in enkele landen het geval. Voor andere vormen is het verschil duidelijker, zeker bij boycots.

- Activisme op het internet: verdeelde meningen. Idee is dat hierdoor de ‘slacktivisten’ meer betrokken kunnen geraken, anderen denken dat het enkel maar de kloof kan verergeren. Cijfers tonen een disproportionele oververtegenwoordiging van hoogopgeleiden (tot 3 maal meer).

- Deliberatieve participatie: publieke dialogen, burgerraden, etc; voice boven vote. Hoe moeilijker de activiteit in kwestie is des te groter de kloof. Hoogopgeleiden zijn vaak retorisch sterker, wat hier hun overwicht kan bepalen. De kloof is groter in bepaalde landen.

IV. Partijlidmaatschap. Daalde sterk in de 20ste eeuw. In het VK heeft de caravanclub meer leden dan alle partijen tezamen. Vooral de hoogopgeleiden bleven lid, maar de kloof is

39

Page 40: Facultair Overlegorgaan Sociale Wetenschappen … · Web viewSolidariteit op de werkvloer maakt deel uit van een ruimere sociale solidariteit 2) Kerk en religiositeit: Georganiseerde

DAAN DELESPAUL 2017-2018

kleiner en soms zelfs onzichtbaar. Er zijn evenwel ook verschillen binnen de partijen zelf te merken. Binnen de FDP en Die Grünen maken de hoogopgeleiden resp. de helft en tweederde van alle leden uit, bij de CDU en de SPD is het verschil minder te zien. Ecolo en D66 zijn de toppers met 75% en 80%. Let op VVD vergeleken met Open VLD en SGP.

The Concentration of Activity

Tabel 5.1 op p. 80 toont de odds-ratio om aan een activiteit deel te nemen voor HO in vergelijking met LO. De ratio is het kleinste voor het dragen van een badge (2,3) en het hoogste voor meedoen in een organisatie of partij (4,6 en 4,9). Dit toont verder een groter verschil voor nieuwe vormen van participatie. Het is ook belangrijk na te gaan hoe vaak iemand participeert, ook hier spannen de hoogopgeleiden de kroon.

Education: a Cause or a Proxy?

We zien dat hogeropgeleiden over alle vlakken meer participeren. De vraag is of scholingsgraad een oorzakelijke factor is of een proxy voor andere factoren zoals inkomen, cognitieve capaciteiten of socialisatiegraad. Doorgaans wordt educatie echter als meest determinerende factor voor participatie gezien. Dit zou ten eerste komen door motivatie (Verba). Onderwijs geeft burgers politieke kennis en interesse en geeft door betere cognitieve capaciteiten de mogelijkheid het proces beter te kunnen analyseren. Ten tweede leert onderwijs een aantal vaardigheden aan die van belang zijn in de politiek. Ten derde stimuleert onderwijs ook sociale participatie en biedt het meer openingen via netwerken. Hogeropgeleiden zijn simpelweg meer geneigd andere hoogopgeleiden te kennen en komen daardoor makkelijker in het politieke proces. De kritiek die onderwijs als een proxy ziet zegt dat men moet focussen op de relatieve en niet absolute cijfers van opleiding. Anderzijds zou het niet de opleiding maar culturele-achtergrond factoren, die zelf onderwijsgraad bepalen, die de participatie bepalen. Iemand met laagopgeleide ouders zou zo zelf ook laagopgeleid worden.

The Paradox of the Participatory Revolution

Dat onderwijs participatie bepaalt is al lang geen geheim meer, maar er is weinig onderzoek gedaan naar de gevolgen hiervan op de democratie. Het is belangrijk om deze variabele als ‘standaard’ te onderzoeken in wetenschappelijk onderzoek, zoals dit voor bv. gender en etniciteit wel is. De reden is waarschijnlijk omdat dit politiek minder zichtbaar is. Meer democratisering zorgt vaak voor een grotere opleidingskloof. Dit zou geen probleem zijn als de hoogopgeleiden de wensen van de laagopgeleiden correct zouden kunnen vertalen. Zoals we uit het inleidende voorbeeld afleidden is dit echter zelden het geval. Anderzijds zouden bepaalde problemen nooit aan de oppervlakte kunnen komen.

HOOFDSTUK 6 – The Meritocratization of Civil Society

The Rise of EU Nerds

De rol van de traditionele middenveldsorganisaties zoals vakbonden en vrouwengilden maakt plaats voor een reeks nieuwe belangengroepen, vaak professioneel gemanaged en werkend met universitair opgeleide consultants. Engagement door de burger heeft zo vaak plaatsgemaakt voor een meritocratie met hoogopgeleiden aan de macht.

Civil Society Organizations

Civil societies wonder vaak gezien als de wonderbaby’s van de democratie: ze versterken sociale cohesie, vertrouwen en de werking van de democratie (Putnam). Ze mobiliseren individuele voorkeuren en geven ze een stem in het publieke debat. De Tocqueville noemde het ‘de school van de democratie’. De civil society is het geheel aan organisaties en instituten tussen staat, markt en gezin. Deze definitie is breed en daarom wordt er vaak een verdere classificatie over twee assen gemaakt. De eerste as peilt naar het doel van de organisatie. Hier onderscheiden we 1) vrijwilligersorganisaties met een neutraal doel (bv. sportclub) 2) sociale belangengroepen met een concreet activistisch doel en 3) non-profitgroepen die binnen een specifieke sector opereren (bv. vakbonden, maar ook denktanks). De tweede as kijkt naar de relatie tussen de organisatie en haar leden. ‘Full participation’ verwijst naar groepen waarbij de leden zelf ook de macht in handen hebben (grassroot). Bij ‘limited participation’-groepen tekenen de leden het beleid en doel uit, maar is er een professionele staff die dat beleid uitwerkt. In ‘token participation’-groepen hebben de leden amper macht en gebeurt alles door de betaalde/ professionele staff.

40

Page 41: Facultair Overlegorgaan Sociale Wetenschappen … · Web viewSolidariteit op de werkvloer maakt deel uit van een ruimere sociale solidariteit 2) Kerk en religiositeit: Georganiseerde

DAAN DELESPAUL 2017-2018

The Context: An Associational Revolution

De civil society werd de afgelopen decennia enorm uitgebouwd. Het zijn vooral de expliciet politieke groepen die toenamen. In West-Europa is rond de 10% van de bevolking in deze sector actief. Ook op EU-niveau neemt het aantal toen, de EU zoekt dan ook naar manieren om het middenveld meer in haar besluitvorming te betrekken. Het Europese Transparantieregister (met verplichte inschrijving voor wie contact met parlementsleden wil) toont dat 50% van de vertegenwoordigde organisaties bij de EU marktgericht zijn. Een op vier betreft NGO’s, 12% is onafhankelijk, burgers zijn dus in de minderheid. De civil society internationaliseert voorts compleet: organisaties zoals Greenpeace en Amnesty International zijn wereldwijd actief. De vraag bij al het voorgaande blijft wel: wie bestuurt deze organisaties?

Participation: the Education Gap in Membership

Het percentage van de bevolking dat lid is van een organisatie is hoger in het noorden van Europa, gevolgd door het zuiden en daarna het oosten. Als we kijken naar de verschillen over scholingsgraad zien we dat de hoogopgeleiden bij elk type organisatie oververtegenwoordigd zijn, behalve vakbonden in België en Nederland en de Kerk. Niet alleen in ledenaantal is een verschil te merken, maar ook in de tijd en (financiële) middelen die men in de organisatie steekt. Ook vrijwilligers zijn doorgaans hoogopgeleid.De opleidingskloof verschilt naargelang het type van organisatie. De grootste kloof is te merken in de politieke activisten- en belangengroepen met humanitaire of milieudoelen. De kleinste kloof bestaat in sportclubs, vakbonden en religieuze instanties. Los van de vakbonden zijn dit geen politieke organisaties, wat dus weer eens bevestigt dat het de hoogopgeleiden zijn die de politieke agenda vooral kunnen beïnvloeden. Vooral civil society II (belangengroepen) kent een sterke kloof, civil society I (vrijwilligers) minder. De nieuwere organisaties vergroten dus de kloof, hoewel die er bij de traditionelere al was.

Transformation: From Participation to Professionalization

De politieke meritocratie laat zich niet enkel merken in een evolutie naar een grote kloof in de leden, maar evenzeer in een evolutie van de organisaties zelf. Door betere financiering worden nieuwe organisaties zo steeds losser van de basis en door een betere beleidsanalyse wordt de beïnvloeding op het politieke proces verhoogt. De arena waarin de organisaties spelen werd steeds competitiever; dit leidde tot een verdere professionalisering en ‘bussinessificatie’. Deze evolutie van massaorganisaties naar professionele belangengroepen zorgde ook voor een verschil in participatievormen. Het aandeel passieve leden wordt steeds groter en hun bijdrage wordt ‘vervangen’ door een betaalde staff. Steeds meer groepen zijn sowieso ‘ledenloos’. Dit is een shift van ‘total’ en ‘limited participation’ naar ‘token participation’. De opkomst van de ‘astroturf participatie’ die gesponsord wordt door machtige belangengroepen en coöperaties wijst verder in de richting van professionalisering. Met astroturven wordt verwezen naar politieke actie die spontaan vanuit de bevolking lijkt te komen, maar uiteindelijk van buiten uit, bijvoorbeeld van een denktank, komt. Deze organisaties houden graag voor uit de bevolking te komen om zo legitimiteit te staven in hun beïnvloeding van het politieke proces. Participatie in een organisatie wordt meer en meer affiliatie. Overheden die de civil society sponsoren om hiermee de participatiedemocratie te verhogen kunnen perverse effecten veroorzaken waardoor de burgers juist meer en meer uit de organisatie verdreven worden, naarmate de financiering onafhankelijker van de basis wordt. Veel groepen proberen het politieke proces niet langer te beïnvloeden door hun leden te mobiliseren, maar veeleer door een technische expertise en lobbying. Dit gaat nu eenmaal veel beter met getraind, professioneel personeel dan met vrijwilligers. Statistisch onderzoek toont aan dat het merendeel van het personeel in deze groepen inderdaad hoogopgeleid is. Het is belangrijk deze evolutie ook in een breder maatschappelijk perspectief te bekijken. De scholingsgraad groeit nu eenmaal, zeker in de dienstensector. De zogenaamde ‘knowledge workers’ hielpen niet enkel met het uitbreiden van de moderne welvaartsstaat, maar evenzeer met de massale uitbreiding van de civil society. Binnen deze civil society zijn het vooral de advocaten en communicatie-experten die aan belang winnen. Een landbouworganisatie aan de EU wordt zo enkel bestuurd door universiteiten, waarvan niemand ooit in het veld heeft gewerkt. Dit noemt men ‘high level unionism’. Tot nu toe hebben de professionals echter de activisten nog niet ingehaald.

The Opening and Closing of the Civil Society

De groei van de civil society heeft haar betekenis evenzeer veranderd. Van de gesloten, op klassen gebaseerde traditionele groepen kwamen er nieuwe, meer open en diverse organisaties. Deze

41

Page 42: Facultair Overlegorgaan Sociale Wetenschappen … · Web viewSolidariteit op de werkvloer maakt deel uit van een ruimere sociale solidariteit 2) Kerk en religiositeit: Georganiseerde

DAAN DELESPAUL 2017-2018

organisaties hebben vooral hoogopgeleide leden, worden geleid door professionals en rekruteren voornamelijk uit diezelfde groep. Ondanks de positieve functie die deze organisaties vaak toegewezen krijgen, blijkt dat slechts uit een kleine poel gevist wordt en een groot deel van de bevolking gerust gelaten wordt. De schools of democracy zijn niet meer wat ze ooit waren. De vraag blijft vooral wat de invloed hiervan is op de democratie. De organisaties zorgen al lang niet meer voor een extra representatieve correctie op het beleid en falen er bovendien in de belangen van de hele bevolking te vertegenwoordigen. Amerikaans onderzoek wijst zo uit dat de thema’s die deze groepen aanhalen vaak afwijken van de thema’s die de bevolking wil, waardoor de samenleving steeds meer naar een oligarchie neigt.

HOOFDSTUK 7 – Political Elites as Educational Elites

Ook hier is een grote kloof, verder niet kennen.

PART III. ConsequencesHOOFDSTUK 8 – The Consequences of Diploma Democracy

Why Bother about Diploma Democracy

Zijn we niet beter af met hoogopgeleide vertegenwoordigers? Plato dacht in ieder geval van wel. In de Republiek beargumenteert hij dat de slimste kinderen van jongs af aan moesten opgeleid worden om uiteindelijk de staat te dienen, zij waren de gouden zielen. De zilveren zielen mochten naar het leger en de bronzen konden boeren worden omdat zij de rationaliteit misten voor een administratie. Michael Young stelde overeenkomstig de formule IQ + ijver = verdienste als basis. Onder dit meritocratisch ideaal moest elk kind een IQ-test ondergaan om in bijpassend onderwijs te worden opgedeeld. Beide systemen faalden, zowel intern als extern, de vraag is dus of meritocratie wel met democratie samengaat.

Representation: Descriptive Representation Matters

De eerste vraag is wie moet vertegenwoordigd worden: het maatschappelijk belang of elk individueel lid van de maatschappij? In het tweede geval gaat een meritocratie dan alvast niet op. Maar is het parlement niet altijd hoogopgeleider geweest dan de samenleving? Onderzoek wijst uit van niet: de onderwijsrevolutie is sneller voltrokken in het parlement dan de samenleving. Sommige onderzoeken spreken dit tegen.De representatie van onderopgeleiden is ten eerste belangrijk omdat zij in een andere sociale wereld leven. Het is bovendien hypocriet wel rekening te houden met andere groepen zoals vrouwen en minderheden maar niet met lageropgeleiden. Dit heeft een symbolisch probleem: het doet uitschijnen dat bepaalde groepen minderwaardig aan anderen zijn en niet competent zouden zijn om hun mening te uiten. Ten tweede is er een democratisch probleem. Kiezer stemmen graag op politici met eenzelfde profiel, bijvoorbeeld wat achtergrond betreft. Dit betekent dat de oververtegenwoordiging simpelweg ongewenst is. Kiezers zijn zelfs vaak geneigd om juist niet op kandidaten met een universitaire opleiding te stemmen. Maar zijn universitairen dan geen betere politici? Onderzoek toont aan dat opgeleide politici niet per se beter hoeven te zijn. Hoogopgeleide Congresleden in de Verenigde Staten keuren niet meer wetten goed, blijven minder lang MP en presteren slechter in de polls. Een Braziliaans onderzoek vond dat hoogopgeleiden vaker corrupt zijn. De afgelopen decennia kwam er een steeds grotere rol voor onverkozen entiteiten zoals (binnen) de EU, civil society organisatie en allerlei andere politieke regulators. Kenmerkend aan deze entiteiten is dat zij opereren in een gesofisticeerde omgeving waardoor ze een gespecialiseerd personeel nodig hebben. Toch kunnen deze instanties een grotere greep op het beleid hebben dan een doorsnee politicus. Daarnaast viel het beleid sowieso al verder uit handen van de politiek door de civil society, die zoals eerder vermeld niet minder ongelijk is. Dit maakt van de politiek steeds meer een aparte sfeer die op vele vlakken los staat van de samenleving.

Responsiveness: Biased Political Agendas

Dan blijft de vraag nog hoe men vertegenwoordigt. Dat een parlementslid niet eenzelfde afkomst heeft betekent daarom nog niet dat hij een persoon niet evengoed kan vertegenwoordigen. Veel socialisten hebben zo een universitaire opleiding genoten, terwijl ze de belangen van de arbeiders even goed verdedigen. Responsiviteit wordt echter problematischer wanneer de verschillende opleidingsniveaus er verschillende politieke meningen op na houden. Veel van deze verschillen in attitudes blijken gesitueerd op de nieuwere breuklijnen. Zo is de kloof bijzonder groot voor socioculturele thema’s zoals immigratie, Europese eenmaking en etnische diversiteit.

42

Page 43: Facultair Overlegorgaan Sociale Wetenschappen … · Web viewSolidariteit op de werkvloer maakt deel uit van een ruimere sociale solidariteit 2) Kerk en religiositeit: Georganiseerde

DAAN DELESPAUL 2017-2018

Hoogopgeleiden worden daarin als ‘kosmopolieten’ beschouwd met pluralistische waarden, laagopgeleiden zijn meer ‘nationalisten’, met een monoculturele maatschappij als streefdoel. Onderzoek wees uit dat de kloof in euroscepticisme in de periode 1973 en 2010 sterk toenam, gaande over thema’s als de euro, het beeld van de EU, optimisme over de EU en het zich zien als EU-burger naast nationale burger. De kloof nam verder toe in 2000 toen de euro werd ingevoerd en is het grootste in Nederland en Denemarken. Ook participatie is verdeeld over scholingsgraad: hoogopgeleiden participeren meer waardoor hun dominerende kosmopolitische waarden een vertekend beeld geven van de maatschappelijke realiteit. De volgende stap is om te kijken naar het beleid: worden hoogopgeleiden ook daarin voorgetrokken door de politiek? Het goede nieuws is dat de responsiviteit tussen burger en politiek fors toenam de afgelopen jaren waardoor de kwaliteit van representatie tegenwoordig op 89% zit. Wanneer men echter op een disaggregaat niveau gaat kijken op basis van onderwijsniveau. Over thema’s zoals multiculturalisme was de responsiviteit voor hoogopgeleiden 94% en voor laagopgeleiden slechts 59%. Verder onderzoek wees wel uit dat de responsiviteit voor lageropgeleiden toenam met de toetreding van rechts-populistische partijen in het parlement. Zoals eerder vermeld zijn er twee stijlen van vertegenwoordiging: mandaat-stijl en trustee-stijl. Het blijkt dat hoogopgeleiden meer te vinden zijn voor een trustee-stijl en laagopgeleiden voor een mandaat-stijl. De parlementsleden zelf zijn duidelijk ook meer voorstander van een trustee-functie, blijkt uit onderzoek. Een verschil in opvattingen over de parlementaire rol is essentieel in een democratie. Eenzelfde, maar kleinere kloof zien we ook terug bij de mening omtrent referenda. Over het algemeen zijn hogeropgeleiden dan ook vaker te vinden voor een democratisch status quo. Over het algemeen kunnen we dus stellen dat de politieke agenda de opvattingen van de hoogopgeleiden over-prioritiseert.

Accountability: Biased Judgement

Door de afwezigheid van laagopgeleiden in parlementen en andere belangengroepen betekent dat zij ook geen rol spelen in het evaluatieproces van het beleid. Ten tweede worden de standaarden die de evaluatie vastleggen door hoogopgeleiden opgesteld, waardoor zij hun focus op de voor hen interessante plaatsten kunnen leggen. De ethische afwegingen van wat wel of niet kan in de politiek ligt dan ook anders naargelang het opleidingsniveau. Laagopgeleiden focussen eerder op wettigheid, hoogopgeleiden willen bovendien ook geen belangenvermenging. Er is zo met andere woorden een verschil tussen wat het brede publiek als standaard aanneemt en wat men in hoogopgeleide kringen van een politicus verwacht. De lageropgeleiden zullen over het algemeen ook een kritischere ethiek ten opzichte van de politiek aannemen.

Legitimacy: Distrust and Resentment

De kans bestaat dat de laagopgeleiden zover van de politiek afdrijven dat ze cynisch worden ten aanzien van het hele systeem. Dit wordt ook wel de ‘disaffectation hypothesis’ genoemd (Young): een laag vertrouwen in de politiek door laagopgeleiden komt door het gevoel uitgesloten te zijn van sociale en politieke participatie. Deze hypothese lijkt te worden bevestigd doordat laagopgeleiden meer aangeven te voelen dat ‘de politiek niet geeft om wat mensen zoals zij denken’, dat men geen invloed heeft op de politiek en dat de meeste politici enkel in het vak zitten voor eigenbelang. Daarom is het politieke vertrouwen in het westen dan ook hoger bij hoogopgeleiden. In Oost-Europa is, door de hoge mate van corruptie, die trend dan weer omgekeerd. Die samenhang met vertrouwen in politiek hangt in het algemeen ook samen met kosmopolitische waarden.De conclusie leidt naar een ‘winner-takes-it-all’-samenleving: degenen met een geprivilegieerde maatschappelijke positie hebben meer jogkansen en meer politieke macht. De diploma democratie zal onder druk komen te staan wanneer degenen in een lagere sociale positie zich niet meer vertegenwoordigd voelen. Door de educatie homogeniteit zullen dezelfde groep deze maatschappij in stand houden en ontstaat er een sociale afsluiting (closure) zoals Lipset en Rokkan beschreven. Populisme als politieke stroming is deels ingebed in de perceptie van een opleidingsconflict. De opkomst van het populisme vormt zo een vuist tegen de klassieke meritocratie en gaat daarom in tegen alles wat zij waardeert.

The Tensions between Meritocracy and Democracy

Plato maakt de vergelijking van de staat als schip. Om het schip te sturen hebben we goede stuurmannen nodig, een meritocratie in plaats van een democratie. Maar Plato vergeet dat de stuurmannen dan niet alleen moeten sturen, maar ook de koers bepalen, terwijl dat de taak van de passagiers is.

HOOFDSTUK 9 – Remedying Diploma Democracy

43

Page 44: Facultair Overlegorgaan Sociale Wetenschappen … · Web viewSolidariteit op de werkvloer maakt deel uit van een ruimere sociale solidariteit 2) Kerk en religiositeit: Georganiseerde

DAAN DELESPAUL 2017-2018

No to the Euro-Academics

Referenda in verband met Europese thema’s spelen zich vaak af op een educatiebreuklijn en niet de traditionele links-rechts breuklijn. De stemmen tegen Europa zijn niet populistisch van aard, maar eerder een protest tegen de ‘verre’ EU die te los van de burger staat. Referenda kunnen zo een actief middel zijn tegen een diploma democratie.

The Rise of Populist Parties: An Increasing Political Visibility of the Less Educated

Populisme hoeft niet als iets negatiefs gezien te worden: het geeft de laagopgeleiden juist de stem die ze lange tijd niet hebben kunnen hebben. In Nederland bleek de culturele positie van de lageropgeleiden zelfs te zijn gestegen nadat partijen als de PVV, TON en LPF het parlement binnenkwamen. Meer zelfs: de standpunten van de lageropgeleiden bleken op bepaalde issues even goed vertegenwoordigd als deze van de hoogopgeleiden. Opvallend is dat meerdere leiders van populistische bewegingen zelf hoogopgeleid zijn (bijvoorbeeld Pim Fortuyn). Van Reybrouck is voorstander van een democratisch populisme: een populisme dat zich afzet van het establishment, maar wel binnen democratische lijnen blijft. Populisme kan van de laagopgeleiden dus een meer politiek zichtbare groep maken.

Spectator Politics: Edyfying the Lesser Educated

Populisme is echter ook gevaarlijk: wat als de populistische partijen besluiten hun kar te keren tegen de democratie? Een andere oplossing, die ook door Plato’s tegenstanders de sofisten werd aangehaald, is de scholingsgraad simpelweg te vergroten en zo elke burger hoogopgeleid te maken. Dit zou hen bovendien meer politieke kennen geven en van hen betere burgers maken. Deze aanpak heeft echter evenzeer nadelen: ten eerste kan er een opleidingsinflatie optreden, als iedereen een hoog diploma haalt zal de lat gewoon elders worden gelegd en komen er nog hoger opgeleiden. Ten tweede blijft de vraag of opleidingsniveau een oorzaak of een proxy is (zie Hoofdstuk 5). Zolang dit debat onopgelost geraakt kan er echter in een tussenoplossing worden voorzien. Een burgerschapsopvoeding in het brede curriculum zou de ‘civic engagement’ en ‘skills’ van de algemene populatie verhogen en kan de negativiteit rond politiek verminderen. Schlozman vond dat dit inderdaad de politieke gelijkheid zou kunnen verhogen, maar dat de voorwaarden daaraan erg specifiek zijn. Soortgelijk onderzoek werd in Nederland gedaan. Wat echter nog belangrijker werd bevonden was om jongeren ook zaken in de politiek te laten doen, hetgeen positiever zou zijn dan leren op zich. Een burgerschapsvak kan echter nooit de enige oplossing zijn.

Participation: Adjusting the Arena

Een ander probleem blijft dan om de laagopgeleiden te laten participeren. Verschillende experimenten, zoals de mini publics waar vergaderingen worden gehouden tussen at random gekozen burgeres en experten, zijn hiervoor reeds bedacht. De deliberatieve peiling (Fishkin) bracht 100 willekeurige burgers samen om te discussiëren over verschillende thema’s met ondersteuning van experten. Vergelijkbaar met de G1000 werd de uitkomst vervolgens besproken waarna een peiling werd afgenomen. In de meeste gevallen werd de over-representatie van de hoogopgeleiden licht weggewerkt, er werden dan ook verschillende initiatieven ondernomen om de laagopgeleiden voldoende te informeren. Desalniettemin bleef de participatiegrens hoog. Burgerjury’s zijn willekeurig samengestelde groepen die zich informeren over een thema en er vervolgens een besluit over nemen via stemming. Dit wordt regelmatig (op lokaal niveau) gebruikt. Om deze initiatieven te laten slagen is het van essentieel belang dat de externe invloed zo min mogelijk doorweegt. Dit kan bijvoorbeeld gebeuren door een grens op het lobbyen te plaatsen en het te reguleren zodat men weet wie elk van hen vertegenwoordigt. Deze regels kunnen evenzeer voor partijen gelden. Een bijkomend probleem blijft de complexiteit van de politiek waardoor het al bijna vereist is een diploma te hebben om alle processen te kunnen opvolgen. Deze vorm van burgerschap is voor vele mensen te veeleisend waardoor de participatie zal tegenvallen. Democratie zou daarom moeten evolueren naar andere modellen, bijvoorbeeld een ‘monitordemocratie’, ‘stealth democracy’ of een ‘audience democracy’. Deze gaan ervan uit dat de burger slechts sporadisch in plaats van continu actief is. En daarom ook sporadisch gaat participeren.

Bringing the Ballot Back in

Meer stemmingen geven meer macht aan laagopgeleiden zolang het principe van ‘one man, one vote’ geldt. Een tool hiervoor kunnen referenda zijn. Zoals eerder aangegeven zijn lageropgeleiden vaak te vinden voor meer directe democratie. Burgers zijn bovendien vaak beter

44

Page 45: Facultair Overlegorgaan Sociale Wetenschappen … · Web viewSolidariteit op de werkvloer maakt deel uit van een ruimere sociale solidariteit 2) Kerk en religiositeit: Georganiseerde

DAAN DELESPAUL 2017-2018

geïnformeerd over politiek wanneer zij uitgebreidere participatierechten hebben. De meeste politicologen staan sceptisch ten aanzien van referenda omdat ze de democratie niet ten goede komen en te veel in het voordeel van populisten speelt. Een tegenvoorbeeld is Zwitserland, waar per jaar rond de vier referenda worden gehouden; toch kent het land een stabiele democratie. Kriesi duidt op de compromiserende functie van referenda in plaats van een polariserende. Echter, het voorbeeld in Hamburg van een paar hoofdstukken geleden toont dat ook referenda niet altijd eerlijk zijn. Lijphart stelde de herinvoering van de stemplicht voor teneinde de kloof te verminderen. Het zou alvast de laagopgeleiden terug naar de stembus halen, of ze dan voor de juiste partij kiezen is een andere zaak. Desondanks zou het de partijen verplichten rekening te houden met de laagopgeleiden aangezien ze niet langer te negeren vallen. De meningen over de stemplicht zijn verdeeld, hoewel de algemene opkomst inderdaad zou stijgen blijft er toch een verschil in opleiding bestaan: de opkomst wordt simpelweg bij elke categorie hoger. Ander onderzoek vond dan weer dat in landen met een lage opkomst de kloof groter is dan landen met een hoog cijfer: een stemplicht zou dan alvast helpen. Ook andere voorstellen in verband met stemmen werden gemaakt: bijvoorbeeld zouden burgers enkel verplicht worden deel te nemen aan hun eerste verkiezingen, wat hen voor later kan socialiseren. Andere auteurs hameren vooral op een gemakkelijkere stemregistratieprocedure, of gewoon een makkelijkere stemming door bijvoorbeeld stemmen via de post of het internet.

More Inclusive Political Elites

Een belangrijke rol is ook weggelegd voor de partijen: zij moeten verder democratiseren zodat laagopgeleiden een even grote kans krijgen. Een open lijst of primaries voor de samenstelling ervan kan daarbij helpen. Ook in hun rekrutering moeten partijen anders te werk gaan en zich meer op laagopgeleiden richten. Vakbonden kunnen hiervoor een belangrijk precedent bieden (bv. in Denemarken veel MPs met lage opleiding met vakbondsachtergrond). Een hardere aanpak kan bestaan in quota’s op de lijsten, vrijgehouden zetels of een voorkeursbehandeling voor laagopgeleiden zoals dit ook vaak het geval is voor vrouwen en minderheden. Deze remedies voor een juiste vertegenwoordiging worden niet door iedereen als de oplossing gezien. Er is geen reden om te denken dat laagopgeleiden meteen de problemen van andere laagopgeleiden in beleid gaan omzetten, net zoals de hoogopgeleide leiders van populistische partijen dat niet voor hoogopgeleiden doen. Met andere woorden representativiteit is niet responsiviteit. Bovendien is er het probleem van essentialisme (als enkel vrouwen vrouwen vertegenwoordigen, waar kan men dan de grens trekken?) en polarisatie tegen de voorkeursbehandeling. Een extreme oplossing zou erin bestaan alle parlementsleden via het lot te verkiezen. In Athene was dit het geval om een meritocratie te vermijden, tegen de zin van Plato. In een afgezwakte versie wordt enkel een senaat hiermee verkozen. De uitvoerende macht is echter best in handen te houden van hoogopgeleiden.De auteurs zijn voorstander van een stemplicht, met een rechtstreekse verkiezing voor de uitvoerende macht. Bovendien moet er een opportuniteitspluralisme komen zodat niet alle kansen vasthangen aan opleiding.

A Wake-Up Call for Modern-Day Platonists

Politiek is te complex, daarom zal een enkele oplossing waarschijnlijk niet werken. Bovendien bestaat er een verschil in het uitvoeren van beleid en het bedenken: dingen kunnen anders uitdraaien. Toch doen politici er beter aan om rekening te houden met de laagopgeleiden, zoals bleek uit recente referenda om de toekomst van Europa.

45